58
Chapter 6 Pythagorean theorem ยฉDavid Morin, October 2021. Draft chapter from eventual book Algebra & Geometry for the Enthusiastic Beginner. Estimated completion date: Fall 2023. More details at https://scholar.harvard.edu/david-morin/books The Pythagorean theorem is one of the most beautiful theorems in mathematics. It is simple to state, easy to use, and highly accessible โ€“ it doesnโ€™t require a huge amount of mathematical machinery to prove. Weโ€™ll be able to prove it (in numerous ways!) with what weโ€™ve learned so far. Weโ€™ll begin by stating the basics of the Pythagorean theorem in Section 6.1, and then in Section 6.2 weโ€™ll discuss the general form of Pythagorean triples, which are triples of integers that satisfy the Pythagorean theorem. Weโ€™ll then prove the theorem in many (seven!) ways in Section 6.3. Some of the proofs require nothing more than the โ„Ž/2 formula for the area of a triangle. And some donโ€™t even require that! Section 6.4 covers an interesting real-life application of the Pythagorean theorem, namely, how far you can see to the horizon from a tall building. Section 6.5 then presents many examples and exercises for practice. Weโ€™ll end with a general discussion in Section 6.6 about the bene๏ฌts of working with letters instead of numbers. 6.1 The theorem The Pythagorean theorem deals with right triangles. To repeat a few things we mentioned in Chapter 5: Right triangles are ones that have a 90 โ—ฆ angle (which is called a โ€œright angleโ€). A 90 โ—ฆ angle is simply what you have at the corner of a rectangle. The two sides that meet at the right angle are perpendicular to each other. These two perpendicular sides in a right triangle are called the legs. The third side (opposite the 90 โ—ฆ angle) is called the hypotenuse. So in Fig. 6.1 the hypotenuse has length , and the legs have lengths and . As with other words 255

Chapter 6 Pythagorean theorem

  • Upload
    others

  • View
    2

  • Download
    0

Embed Size (px)

Citation preview

Chapter 6

Pythagorean theoremยฉDavid Morin, October 2021.Draft chapter from eventual book Algebra & Geometry for the Enthusiastic Beginner. Estimatedcompletion date: Fall 2023. More details at https://scholar.harvard.edu/david-morin/books

The Pythagorean theorem is one of the most beautiful theorems in mathematics.It is simple to state, easy to use, and highly accessible โ€“ it doesnโ€™t require a hugeamount of mathematical machinery to prove. Weโ€™ll be able to prove it (in numerousways!) with what weโ€™ve learned so far.

Weโ€™ll begin by stating the basics of the Pythagorean theorem in Section 6.1, andthen in Section 6.2 weโ€™ll discuss the general form of Pythagorean triples, which aretriples of integers that satisfy the Pythagorean theorem. Weโ€™ll then prove the theoremin many (seven!) ways in Section 6.3. Some of the proofs require nothing more thanthe ๐‘โ„Ž/2 formula for the area of a triangle. And some donโ€™t even require that!Section 6.4 covers an interesting real-life application of the Pythagorean theorem,namely, how far you can see to the horizon from a tall building. Section 6.5then presents many examples and exercises for practice. Weโ€™ll end with a generaldiscussion in Section 6.6 about the benefits of working with letters instead ofnumbers.

6.1 The theorem

The Pythagorean theorem deals with right triangles. To repeat a few things wementioned in Chapter 5: Right triangles are ones that have a 90โ—ฆ angle (which iscalled a โ€œright angleโ€). A 90โ—ฆ angle is simply what you have at the corner of arectangle. The two sides that meet at the right angle are perpendicular to eachother. These two perpendicular sides in a right triangle are called the legs. Thethird side (opposite the 90โ—ฆ angle) is called the hypotenuse. So in Fig. 6.1 thehypotenuse has length ๐‘, and the legs have lengths ๐‘Ž and ๐‘. As with other words

255

256 Chapter 6. Pythagorean theorem

like โ€œradiusโ€ and โ€œcircumference,โ€ the words โ€œhypotenuseโ€ and โ€œlegโ€ can refer toeither the segment itself (as in the preceding sentence), or the length of the segment(as in โ€œthe hypotenuse is ๐‘โ€). The usage is generally clear from the context. Thestandard notation for a right angle is a little square, as we have drawn.

a

b

c

Figure 6.1

As mentioned in Section 5.4, the Pythagorean theorem states that the sides of aright triangle are related by

๐‘Ž2 + ๐‘2 = ๐‘2 (Pythagorean theorem) (6.1)

This statement of the Pythagorean theorem was certainly known before Pythagorasโ€™time, although it is unknown how much earlier. The date (and creator) of the firstproof is also unknown. In any case, we can only wonder what Pythagorasโ€™ firstencounter with the theorem looked like. . .

Pythagoras wept and despairedAs he added the legs and compared.Then he jumped up with glee,โ€œThough they donโ€™t add to ๐‘,Itโ€™s a match if the lengths are all squared!โ€

Weโ€™ll prove the theorem in Section 6.3 below, but there are a few things weshould discuss first. If you draw a triangle with a random shape, the odds are that itwonโ€™t be a right triangle. That is, most random sets of three numbers ๐‘Ž, ๐‘, ๐‘ donโ€™tsatisfy Eq. (6.1). Only special sets do, and hence yield a right triangle. The simplestset of integers that satisfy the theorem is 3, 4, 5. These lengths produce a righttriangle because

32 + 42 = 52 โ‡โ‡’ 9 + 16 = 25. (6.2)

Most right triangles donโ€™t have integer lengths for all three sides. Or said in anotherway, if you pick integers for two sides of a right triangle, the third side probablywonโ€™t be an integer. For example, if we pick the two legs to be 1 and 1, then thehypotenuse is given by

12 + 12 = ๐‘2 =โ‡’ ๐‘2 = 2 =โ‡’ ๐‘ =โˆš

2 โ‰ˆ 1.414, (6.3)

6.1. The theorem 257

which isnโ€™t an integer. (This triangle is our old friend, the 45-45-90 right triangle.)Or if we pick the hypotenuse to be 8 and one leg to be 5, then the other leg is givenby

๐‘Ž2 + 52 = 82 =โ‡’ ๐‘Ž2 + 25 = 64. (6.4)

Subtracting 25 from both sides of this equation (as we learned in Section 4.5), andthen taking the square root of both sides, gives

๐‘Ž2 = 39 =โ‡’ ๐‘Ž =โˆš

39 โ‰ˆ 6.245, (6.5)

which isnโ€™t an integer.If all three sides of a right triangle are integers, then we call the set of these

integers a Pythagorean triple (or just a triple, for short). People often list theintegers of a triple inside parentheses, like โ€œ(๐‘Ž, ๐‘, ๐‘).โ€ For example, in addition tothe Pythagorean triple (3, 4, 5) mentioned above, a few other triples are (6, 8, 10),(5, 12, 13), and (8, 15, 17) because, as you can verify,

62 + 82 = 102, 52 + 122 = 132, 82 + 152 = 172. (6.6)

A quick way of producing new triples from other known triples is to use the factthat any integer multiple of the three numbers in a triple yields three new numbersthat are again a triple. This is true because if (๐‘Ž, ๐‘, ๐‘) is a triple, then we can multipleboth sides of the Pythagorean theorem by ๐‘ 2 (where ๐‘  is an integer) to obtain anothertrue statement. (If the two sides of an equation are equal, then multiplying thesetwo equal quantities by the same number ๐‘ 2 yields two new quantities that are againequal.) This multiplication by ๐‘ 2 yields

๐‘Ž2 + ๐‘2 = ๐‘2 =โ‡’ ๐‘ 2๐‘Ž2 + ๐‘ 2๐‘2 = ๐‘ 2๐‘2 =โ‡’ (๐‘ ๐‘Ž)2 + (๐‘ ๐‘)2 = (๐‘ ๐‘)2. (6.7)

But this is just the statement that (๐‘ ๐‘Ž, ๐‘ ๐‘, ๐‘ ๐‘) is a Pythagorean triple, as we wantedto show. For example, the (6, 8, 10) triple mentioned above is the (3, 4, 5) triplemultiplied by ๐‘  = 2.

This multiplication of each side of a right triangle by ๐‘  and ending up withanother right triangle makes intuitive sense. If youโ€™re given a right triangle, and ifyou scale it up uniformly by multiplying all of the sides by the same factor, thenthe new triangle has the same shape as the old one, so itโ€™s still a right triangle. Thenew triangle is similar to the old one (it has the same shape); recall the discussionof similarity in Section 5.4. Even if ๐‘  isnโ€™t an integer, weโ€™ll still end up with a righttriangle. But if the sides arenโ€™t integers, we donโ€™t call it a Pythagorean triple.

Note that the Pythagorean theorem in Eq. (6.1) is symmetric in ๐‘Ž and ๐‘. Thatis, both ๐‘Ž and ๐‘ are raised to the same power (namely 2), and the two terms havethe same coefficient (namely 1). This symmetry follows from the fact that it canโ€™tmatter which leg you arbitrarily choose to label as ๐‘Ž, and which one you label as

258 Chapter 6. Pythagorean theorem

๐‘. If someone claimed that the theorem took the form of, say, ๐‘Ž2 + 2๐‘2 = ๐‘2, thenyou would get a different result for ๐‘ if you switched your ๐‘Ž and ๐‘ labels. So thisโ€œtheoremโ€ canโ€™t be correct.

For example, if the two legs are 5 and 8, and if we label them as ๐‘Ž = 5and ๐‘ = 8 (which weโ€™re free to do), then the ๐‘Ž2 + 2๐‘2 = ๐‘2 โ€œtheoremโ€ gives52 + 2 ยท 82 = ๐‘2 =โ‡’ 153 = ๐‘2 =โ‡’ ๐‘ =

โˆš153 = 12.4. However, if we label

the legs as ๐‘Ž = 8 and ๐‘ = 5 (which weโ€™re also free to do), the โ€œtheoremโ€ gives82 + 2 ยท 52 = ๐‘2 =โ‡’ 114 = ๐‘2 =โ‡’ ๐‘ =

โˆš114 = 10.7. But there can be only one

value of ๐‘, of course. So the fact that our formula gives two different values meansit canโ€™t be correct.

6.2 General form of triples

It turns out that there is a very simple and general way to produce Pythagoreantriples, beyond the easy ones that are simply integer multiples of other triples. Weclaim that if we start with any two integers ๐‘š and ๐‘›, then the following three integers๐‘Ž, ๐‘, ๐‘ are a Pythagorean triple, that is, they satisfy the Pythagorean theorem:

๐‘Ž = ๐‘š2 โˆ’ ๐‘›2, ๐‘ = 2๐‘š๐‘›, ๐‘ = ๐‘š2 + ๐‘›2. (6.8)

You can verify this claim by doing the following exercise.

Exercise 6.1 Show that the ๐‘Ž, ๐‘, and ๐‘ expressions in Eq. (6.8) satisfyEq. (6.1), by calculating the sum ๐‘Ž2 + ๐‘2 and then showing that the resultequals ๐‘2.

For integers ๐‘š, ๐‘›, who knewThat ๐‘ is their product times 2?And ๐‘Ž? Itโ€™s a fact:Form the squares and subtract.And then ๐‘? Instead add up the two.

The preceding exercise is the standard way of showing that the ๐‘Ž, ๐‘, and ๐‘

expressions in Eq. (6.8) form a Pythagorean triple. Hereโ€™s another way. We wantto show that ๐‘Ž2 + ๐‘2 = ๐‘2, and this relation is equivalent to (by subtracting ๐‘Ž2 fromboth sides) ๐‘2 = ๐‘2 โˆ’ ๐‘Ž2. We can now invoke the handy difference-of-squares result

6.2. General form of triples 259

from Eq. (3.22) to write ๐‘2 โˆ’ ๐‘Ž2 as (๐‘ + ๐‘Ž) (๐‘ โˆ’ ๐‘Ž). So our goal is to show that thisproduct equals ๐‘2. Plugging in the expressions for ๐‘Ž and ๐‘ from Eq. (6.8) gives

๐‘2 โˆ’ ๐‘Ž2 = (๐‘ + ๐‘Ž)(๐‘ โˆ’ ๐‘Ž)

=((๐‘š2 +๏ฟฝ๏ฟฝ๐‘›

2) + (๐‘š2 โˆ’๏ฟฝ๏ฟฝ๐‘›2)

) ((๏ฟฝ๏ฟฝ๐‘š2 + ๐‘›2) โˆ’ (๏ฟฝ๏ฟฝ๐‘š2 โˆ’ ๐‘›2)

)= (2๐‘š2) (2๐‘›2) = 4๐‘š2๐‘›2 = (2๐‘š๐‘›)2 = ๐‘2, (6.9)

as desired.

Exercise 6.2 Show again that the ๐‘Ž, ๐‘, and ๐‘ expressions in Eq. (6.8) satisfyEq. (6.1), by applying the difference-of-squares result like we just did, butnow with the Pythagorean theorem written as ๐‘Ž2 = ๐‘2 โˆ’ ๐‘2.

It turns out that not only does Eq. (6.8) generate Pythagorean triples, it generatesall of them. That is, there are no triples that arenโ€™t of the form in Eq. (6.8); everytriple has an associated (๐‘š, ๐‘›) pair. The proof of this statement (โ€œIf three numbersare a triple, then they take the form of Eq. (6.8)โ€) is more involved than our aboveproofs of the reverse statement (โ€œIf three numbers take the form of Eq. (6.8), thenthey are a tripleโ€). So weโ€™ll just accept it here.

Table 6.1 lists the triples that Eq. (6.8) generates for various (๐‘š, ๐‘›) pairs. Someof the triples are multiples of others. For example, (24, 10, 26) is 2 times (5, 12, 13),although in a different order.

๐‘Ž ๐‘ ๐‘

๐‘š ๐‘› ๐‘š2 โˆ’ ๐‘›2 2๐‘š๐‘› ๐‘š2 + ๐‘›2

2 1 3 4 53 1 8 6 103 2 5 12 134 1 15 8 174 2 12 16 204 3 7 24 255 1 24 10 265 2 21 20 295 3 16 30 345 4 9 40 41

Table 6.1: Pythagorean triples

260 Chapter 6. Pythagorean theorem

Exercise 6.3 Pick a few of the triples in Table 6.1 and verify that they doindeed satisfy the Pythagorean theorem.

If you stare at Table 6.1 long enough, a few things become clear, one of whichis the following. Look at the cases where ๐‘› = ๐‘š โˆ’ 1. So (๐‘š, ๐‘›) takes the formof (2, 1), (3, 2), (4, 3), (5, 4), etc. The ๐‘Ž values associated with these pairs are,respectively, the odd numbers 3, 5, 7, and 9, which equal ๐‘š + ๐‘›. And in each case,you will observe that ๐‘ and ๐‘ differ by 1 and add up to ๐‘Ž2. For example, in the(5, 4) case we have 40 + 41 = 92. And in the (4, 3) case we have 24 + 25 = 72. Thefollowing example shows that this pattern holds for all of the (๐‘š, ๐‘›) = (๐‘š, ๐‘š โˆ’ 1)cases.

Example 6.1 For the ๐‘› = ๐‘š โˆ’ 1 cases, show that ๐‘Ž is odd and equals ๐‘š + ๐‘›.And show that ๐‘ and ๐‘ differ by 1 and add up to ๐‘Ž2.

Solution: If we plug ๐‘› = ๐‘šโˆ’1 into the expressions for ๐‘Ž, ๐‘, and ๐‘ in Eq. (6.8),we obtain

๐‘Ž = ๐‘š2 โˆ’ (๐‘š โˆ’ 1)2 =๏ฟฝ๏ฟฝ๐‘š2 โˆ’ (๏ฟฝ๏ฟฝ๐‘š2 โˆ’ 2๐‘š + 1) = 2๐‘š โˆ’ 1,๐‘ = 2๐‘š(๐‘š โˆ’ 1) = 2๐‘š2 โˆ’ 2๐‘š,

๐‘ = ๐‘š2 + (๐‘š โˆ’ 1)2 = ๐‘š2 + (๐‘š2 โˆ’ 2๐‘š + 1) = 2๐‘š2 โˆ’ 2๐‘š + 1. (6.10)

We want to show four things:

โ€ข ๐‘Ž is odd: Since ๐‘Ž takes the form of 2๐‘š โˆ’ 1 where ๐‘š is an integer, wesee that ๐‘Ž is indeed odd. (Even numbers take the form of 2๐‘š, and oddnumbers take the form of 2๐‘š โˆ’ 1. Or equivalently 2๐‘š + 1.)

โ€ข ๐‘Ž = ๐‘š + ๐‘›: Since ๐‘š + ๐‘› = ๐‘š + (๐‘š โˆ’ 1) = 2๐‘š โˆ’ 1, we see that ๐‘Ž is equalto ๐‘š + ๐‘›, as desired. Alternatively, the difference-of-squares result inEq. (3.22) tells us that if ๐‘› = ๐‘š โˆ’ 1, then

๐‘Ž = ๐‘š2 โˆ’ ๐‘›2 = (๐‘š + ๐‘›) (๐‘š โˆ’ ๐‘›) = (๐‘š + ๐‘›) (1) = ๐‘š + ๐‘›, (6.11)

because the difference between ๐‘š and ๐‘› is 1.

โ€ข ๐‘ and ๐‘ differ by 1: The forms we found for ๐‘ and ๐‘ in Eq. (6.10) tell usthat ๐‘ = ๐‘ + 1, so they do in fact differ by 1.

6.2. General form of triples 261

โ€ข ๐‘ + ๐‘ = ๐‘Ž2: The sum of ๐‘ and ๐‘ is

๐‘ + ๐‘ = (2๐‘š2 โˆ’ 2๐‘š) + (2๐‘š2 โˆ’ 2๐‘š + 1) = 4๐‘š2 โˆ’ 4๐‘š + 1. (6.12)

This is indeed equal to ๐‘Ž2 since

๐‘Ž2 = (2๐‘š โˆ’ 1)2 = 4๐‘š2 โˆ’ 4๐‘š + 1. (6.13)

Here are a few exercises, two involving numbers and two involving letters.

Exercise 6.4 The size of a rectangular computer screen is generally specifiedby giving the length of the diagonal line. What is the size of a screen that is11.3 inches wide and 7.0 inches tall? Or 14.4 inches wide and 9.0 inches tall?

Exercise 6.5 An American football field is 100 yards long (excluding theend zones) and 53.33 yards (160 feet) wide. If you walk from one corner tothe opposite one, what distance do you save by walking diagonally instead ofalong two sides?

Exercise 6.6 In Table 6.1, another thing you may have noticed is that for the๐‘› = 1 cases, ๐‘Ž and ๐‘ are obtained by taking half of ๐‘, squaring the result, andthen adding or subtracting 1. For example, in the (5, 1) case, ๐‘ is 10. Half ofthis is 5, and 52 is 25. Adding or subtracting 1 then gives 24 and 26, whichare indeed the ๐‘Ž and ๐‘ values. The concise way of stating this result is that ๐‘Žand ๐‘ take the form of (๐‘/2)2 ยฑ 1. Explain why this is true by letting ๐‘› = 1 inEq. (6.8).

Exercise 6.7 Another way of stating the ๐‘› = ๐‘š โˆ’ 1 result discussed abovein Example 6.1 is the following. Take an odd number (call it ๐‘Ž) and squareit. Then add or subtract 1 from the result. And then divide each of thesetwo numbers by 2. Call the results ๐‘ and ๐‘. (So if we start with ๐‘Ž = 9, weobtain 81, and then 82 and 80, and then 41 and 40. This is the last triple inTable 6.1.)

For an (๐‘Ž, ๐‘, ๐‘) triplet generated this way, calculate the sum ๐‘Ž2 + ๐‘2 and thenshow that it equals ๐‘2. Hint: Since ๐‘Ž is an odd number, it takes the generalform of 2๐‘˜ โˆ’ 1 where ๐‘˜ is an integer. (This problem gets a little messy. Youโ€™llneed to square a trinomial, but stick with it!)

262 Chapter 6. Pythagorean theorem

6.3 Seven proofs of the theorem

Letโ€™s now prove the Pythagorean theorem โ€“ in seven different ways! Weโ€™ll present allof these proofs as exercises, so that youโ€™ll have the chance to do them yourself. Wedonโ€™t want you to miss out on any of the fun! Some of the proofs are great examplesof what algebra can do for you, while others (the third and fourth ones) are purelygeometric and donโ€™t require any algebra at all. You might think itโ€™s excessive toinclude seven proofs here, but the strategies involved are varied and instructive. Andbesides, itโ€™s always hard to pass up any new opportunity to prove the Pythagoreantheorem! Hereโ€™s the statement of the theorem:

Pythagorean theorem: If the sides of a right triangle are ๐‘Ž, ๐‘, and ๐‘, with ๐‘ beingthe hypotenuse, then ๐‘Ž2 + ๐‘2 = ๐‘2.

Proof: And here are the proofs:

Exercise 6.8 (Proof 1) Prove the Pythagorean theorem by using the fact thatthe area of the overall square in Fig. 6.2 equals the sum of the areas of thefour triangles plus the area of the smaller square. (Thereโ€™s a bit of an opticalillusion in this figure. The sides of the overall square are indeed vertical andhorizontal, even if they donโ€™t look it!)

a

a

bc

Figure 6.2

Exercise 6.9 (Proof 2) Fig. 6.3 shows a square with side length ๐‘ subdividedinto four right triangles with legs ๐‘Ž and ๐‘ (and hypotenuse ๐‘), along with asquare in the middle with side length ๐‘ โˆ’ ๐‘Ž. Prove the Pythagorean theoremby using the fact that (as in the preceding proof) the area of the overall squareequals the sum of the areas of the four triangles plus the area of the smallersquare.

6.3. Seven proofs of the theorem 263

a

a

b

b โˆ’ a

b โˆ’ a

b

c

c

Figure 6.3

Exercise 6.10 (Proof 3) This proof requires no algebra; itโ€™s basically ageometry-only interpretation of the first proof above. Perhaps it shouldnโ€™tcount as a separate proof, but itโ€™s so slick, I think it should. Itโ€™s the quickestand simplest proof of them all.

Fig. 6.4 shows four shaded triangles inside a square. Show how to rearrangethe triangles in a way that makes it clear that the area of the white region(which is ๐‘2) also equals ๐‘Ž2 + ๐‘2.

b

b

c

a

a

c2

Figure 6.4

Exercise 6.11 (Proof 4) Hereโ€™s another geometry-only proof. Fig. 6.5 showsa combo version of Figs. 6.2 and 6.3. Rearrange some of the shapes to showthat ๐‘Ž2 + ๐‘2 = ๐‘2. Weโ€™ve given a hint by drawing two shaded squares withareas ๐‘Ž2 and ๐‘2. (These shapes are indeed squares, since all sides are either๐‘Ž or ๐‘.)

Exercise 6.12 (Proof 5) The overall right triangle in Fig. 6.6 has side lengths๐‘Ž, ๐‘, and ๐‘. The altitude to the hypotenuse is drawn. Explain why the twosmaller right triangles produced are similar to (that is, they have the same

264 Chapter 6. Pythagorean theorem

a

a

a

b

b

b c

c

Figure 6.5

angles as) the overall right triangle. Then use this similarity to find the twolengths that ๐‘ is divided into. The Pythagorean theorem will follow from thefact that these two lengths add up to ๐‘.

a

b

c

Figure 6.6

Exercise 6.13 (Proof 6) In the preceding proof, we found that the two sub-triangles in Fig. 6.6 are similar to the overall triangle. This similarity allowsus to use the scaling results from Section 5.5 to determine how the areas of thesub-triangles are related to the area of the overall triangle. The Pythagoreantheorem will follow from the fact that the two sub-areas add up to the overallarea.

Exercise 6.14 (Proof 7) This final proof is how Euclid proved the Pythagoreantheorem. Itโ€™s a bit more involved than the preceding six proofs, and it relieson one fact that we havenโ€™t covered yet โ€“ the (entirely believable) โ€œside-angle-sideโ€ (SAS) postulate, which says that if two triangles have two sides incommon, along with the angle between them, then they are congruent (whichis a fancy word for identical; they have the same shape and same size). If youplay around with a few examples, youโ€™ll be convinced that this postulate iscorrect.

6.3. Seven proofs of the theorem 265

In Fig. 6.7, we have drawn squares on the sides of right triangle ๐ด๐ต๐ถ. Provingthe Pythagorean theorem is equivalent to showing that the sum of the areasof the two smaller squares (๐‘Ž2 + ๐‘2) equals the area of the big square (๐‘2).

Your task: First show that the triangles in each shaded pair in Fig. 6.7 arecongruent, and then explain how their areas relate to the areas of the smallersquares, and also to the two rectangular sub-areas ๐‘…1 and ๐‘…2 of the largesquare, defined by the dashed line drawn. Hint: The area of a triangle is halfthe base times the height. Find some helpful bases and heights! For example,triangle ๐ด1๐ต๐ด can be considered to have base ๐ด1๐ต and height ๐ด1๐ด2.

a

a

b

b

A2

A1

C1

C2

R2

R1

B1 B2

C

B

A

cc

c

Figure 6.7

266 Chapter 6. Pythagorean theorem

Having worked through all of these proofs, you can now say without a doubtthat you understand the Pythagorean theorem! And that has its perks. . .

Itโ€™s quick to spot which kids are cool,As they saunter the halls of the school.โ€œWhoโ€™s got the swagger? Us!We know Pythagoras!Sure, weโ€™re all square, but we rule!โ€

The converse

The Pythagorean theorem says, โ€œIf a triangle is right, then its side lengths satisfy๐‘Ž2 + ๐‘2 = ๐‘2.โ€ The reverse statement (the converse) is also true:

Converse: If a triangleโ€™s side lengths satisfy ๐‘Ž2 + ๐‘2 = ๐‘2, then the triangle is right.

As with the โ€œforwardโ€ direction of the theorem we proved above, there are manydifferent ways to prove the converse. Weโ€™ll present just one proof here.

Weโ€™re starting with the assumption that ๐‘Ž2 + ๐‘2 = ๐‘2, and our goal is to showthat the triangle is right. That is, we want to show that it cannot look like either ofthe triangles (obtuse or acute) in Fig. 6.8, where the ๐‘ side is tilted. So for both ofthese possibilities, our goal is to show that ๐‘ฅ must be zero. That is, the ๐‘ฅ segmentmust in fact not exist; equivalently the top vertex is actually directly over the left endof the ๐‘Ž side. Weโ€™ll address the obtuse case here. (The acute case proceeds in thesame manner, with only one small sign modification, as you can check.) This proofmakes use of the โ€œforwardโ€ direction of the Pythagorean theorem, so it assumes(quite correctly!) that weโ€™ve already proved that.

a a

b2 โˆ’ x2

x x

bc

(a)(obtuse) (acute)

(b)

bc

b2 โˆ’ x2

Figure 6.8

From the Pythagorean theorem, the vertical leg of the small right triangle inFig. 6.8(a) has length

โˆš๐‘2 โˆ’ ๐‘ฅ2, as shown. The Pythagorean theorem applied to the

6.4. Distance to the horizon 267

overall big right triangle then gives

(๐‘Ž + ๐‘ฅ)2 +(โˆš

๐‘2 โˆ’ ๐‘ฅ2)2 = ๐‘2

=โ‡’ (๐‘Ž2 + 2๐‘Ž๐‘ฅ +๏ฟฝ๏ฟฝ๐‘ฅ2) + (๐‘2 โˆ’๏ฟฝ๏ฟฝ๐‘ฅ

2) = ๐‘2. (6.14)

Subtracting ๐‘2 from both sides yields

(๐‘Ž2 + ๐‘2 โˆ’ ๐‘2) + 2๐‘Ž๐‘ฅ = 0 =โ‡’ 0 + 2๐‘Ž๐‘ฅ = 0, (6.15)

where we have used the given information that ๐‘Ž2 + ๐‘2 = ๐‘2 =โ‡’ ๐‘Ž2 + ๐‘2 โˆ’ ๐‘2 = 0.We see that the product 2๐‘Ž๐‘ฅ equals zero. And since neither 2 nor ๐‘Ž is zero, it mustbe the case that ๐‘ฅ = 0. In other words, the ๐‘ side is vertical, and the triangle is aright triangle, as we wanted to show.

6.4 Distance to the horizon

Hereโ€™s an interesting real-life application of the Pythagorean theorem: If youโ€™re ina tall building with height โ„Ž, how far can you see to the horizon? Letโ€™s assume thatthe building is at the ocean shore, so that we donโ€™t need to worry about trees andhills and such.

The basic setup is shown in Fig. 6.9. The height โ„Ž weโ€™ve drawn is very muchexaggerated (being about 1/5 of the earthโ€™s radius ๐‘…), to make it easier to see whatโ€™sgoing on. In reality, thereโ€™s no chance that โ„Ž (for any everyday-type scenario) wouldbe comparable to the earthโ€™s radius ๐‘…. Even at the altitude of the International SpaceStation (which is about 250 miles), โ„Ž is only 1/16 of ๐‘… (which is about 4000 miles).

R

R

hd

you

you can see to here

Figure 6.9

Our goal is to find the distance ๐‘‘ in the figure, in terms of the known quantitiesโ„Ž and ๐‘…. Now, the desired distance you can see to the horizon is slightly ambiguous.Do we mean the straight-line distance ๐‘‘ drawn, or do we mean the curved distancealong the surface of the earth? Fortunately it doesnโ€™t matter, because these twodistances are essentially equal for any reasonable (not excessively large) value of โ„Ž.But for concreteness letโ€™s say that our goal is to find the straight-line distance ๐‘‘.

268 Chapter 6. Pythagorean theorem

The straight line representing the distance ๐‘‘ is tangent to the earth. Weโ€™ll talkabout โ€œtangentsโ€ below in Exercise 6.24, but in short, a tangent line is one that justbarely skims the circle. The tangent line is in fact the line of sight weโ€™re concernedwith, because if you look at an angle that is slightly too high, youโ€™ll be looking ata point in the sky; and if you look at an angle that is slightly too low, youโ€™ll belooking at a nearby point on the ground (which therefore wonโ€™t be the farthest pointyou can see). So we are indeed concerned with the tangent line โ€“ the cutoff betweenthe ground and the sky. Weโ€™ll see in Exercise 6.24 that a tangent line is alwaysperpendicular to the radius at the point of contact with the circle. Letโ€™s just acceptthis (quite believable) fact for now. We therefore have the right triangle shown inFig. 6.9, which means that we can apply the Pythagorean theorem.

Letโ€™s do a numerical example first, and then weโ€™ll find the general solution interms of letters. Weโ€™ll pick โ„Ž to be

โ„Ž = 100 meters, (6.16)

which is about 330 feet โ€“ a reasonably tall building. We could work with any unitof length (feet, yards, meters, etc.), but weโ€™ll choose the metric systemโ€™s metersbecause other lengths in it (like kilometers) are obtained by multiplying by simplepowers of 10. A kilometer is 1000 meters. (The prefix โ€œkiloโ€ means 1000.) Theabbreviations for meter and kilometer are โ€œmโ€ and โ€œkm.โ€ A meter is about 39.4inches, which is a little more that a yard (3 feet, or 36 inches).

In addition to assuming weโ€™re at the ocean shore (so that we donโ€™t need to worryabout trees and hills), weโ€™ll work in the approximation where the earth is a perfectsphere. It actually isnโ€™t; it bulges a little at the equator because itโ€™s spinning. Theradius of the earth varies from about 6,356 km at the poles to 6,378 km at the equator.Thereโ€™s no need for that level of accuracy here, so weโ€™ll just round these values upto 6,400 km. Hence

๐‘… = 6,400 km (or equivalently 6,400,000 meters), (6.17)

which is the same as the 4000 miles we stated above. This follows from the factthat there are 1609 meters in a mile, and hence about 1.6 kilometers in a mile (amile is the larger of the two units). So to go from km to miles, you divide by1.609, or equivalently multiply by 0.62. (A 10 km race is 6.2 miles.) This checks:(6400) (0.62) โ‰ˆ 4000, and (4000) (1.6) = 6400.

We can now apply the Pythagorean theorem. In Fig. 6.9, the three sides ofour right triangle are the desired distance ๐‘‘, the radius ๐‘… = 6,400,000 m, and thehypotenuse ๐‘… + โ„Ž = 6,400,100 m. The Pythagorean theorem therefore gives

๐‘‘2 + (6,400,000 m)2 = (6,400,100 m)2. (6.18)

Subtracting (6,400,000 m)2 from both sides of this equation gives

๐‘‘2 = (6,400,100 m)2 โˆ’ (6,400,000 m)2 = 1,280,010,000 m2. (6.19)

6.4. Distance to the horizon 269

Taking the square root of both sides then gives

๐‘‘ =โˆš

1,280,010,000 m2 โ‰ˆ 35,800 m โ‰ˆ 36 km. (6.20)

You can therefore see about 36 kilometers (or about (36)(0.62) = 22 miles) from a100-meter building. Thatโ€™s quite far!

Using letters

You undoubtedly noticed that the above calculation contained some large numbers,which were somewhat of a pain. The numbers would have been smaller if we hadchosen to work with kilometers instead of meters (the lengths would have been๐‘… = 6400 km and ๐‘… + โ„Ž = 6400.1 km), but numbers are still often a hassle to workwith. So letโ€™s now solve the problem algebraically, that is, in terms of letters. Thereare significant advantages to working with letters, as weโ€™ll spell out in Section 6.6.

In terms of letters, applying the Pythagorean theorem to the triangle in Fig. 6.9gives (in place of Eq. (6.18) with numbers)

๐‘‘2 + ๐‘…2 = (๐‘… + โ„Ž)2 =โ‡’ ๐‘‘2 +๏ฟฝ๏ฟฝ๐‘…2 =๏ฟฝ๏ฟฝ๐‘…

2 + 2๐‘…โ„Ž + โ„Ž2

=โ‡’ ๐‘‘ =โˆš

2๐‘…โ„Ž + โ„Ž2, (6.21)

where we have subtracted ๐‘…2 from both sides, and then taken the square root ofboth sides, to obtain the last line. This

โˆš2๐‘…โ„Ž + โ„Ž2 result is the general answer to

the problem. For any values of ๐‘… and โ„Ž weโ€™re given, we can simply plug theminto

โˆš2๐‘…โ„Ž + โ„Ž2 to obtain the desired distance ๐‘‘. You can check that if you plug

in the ๐‘… = 6,400,000 m and โ„Ž = 100 m values we used above, you will reproduceEq. (6.20).

Eq. (6.21) is a nice clean result. But we can go one step further to obtain aneven cleaner result. In a real-life situations, the โ„Ž2 term is much smaller than the2๐‘…โ„Ž term. It is smaller by the factor โ„Ž/2๐‘… (since 2๐‘…โ„Ž ยท โ„Ž/2๐‘… = โ„Ž2), which is verysmall for any everyday value of โ„Ž. If youโ€™re in a tall building with height โ„Ž = 100 m(330 feet), then

โ„Ž

2๐‘…=

100 m2(6,400,000 m) =

1128,000

โ‰ˆ 8 ยท 10โˆ’6. (6.22)

Even at the height of a commercial airplane (about 10,000 m, or 33,000 feet), thevalue of โ„Ž/2๐‘…, which is 100 times larger than for 100 m (or 330 feet), is still only1/1280. So to a good approximation we can simply ignore the โ„Ž2 term in Eq. (6.21)and say that

๐‘‘ โ‰ˆโˆš

2๐‘…โ„Ž. (6.23)

This is an extremely clean result! Now, whenever you derive an approximateanswer like we just did, you gain something and you lose something. You lose

270 Chapter 6. Pythagorean theorem

some truth, of course, because your new answer is an approximation and thereforetechnically not correct (although the error becomes very small in the appropriatelimit โ€“ small โ„Ž here). But you gain some aesthetics. Your new answer is invariablymuch cleaner (often involving only one term), which makes it much easier to seewhatโ€™s going on.

For example, a quick look at Eq. (6.23) tells you that ๐‘‘ does not grow linearlywith โ„Ž (that is, it isnโ€™t directly proportional to โ„Ž), but instead grows like the squareroot of โ„Ž. So if you want to make ๐‘‘ be, say, 5 times larger, then you need to makeโ„Ž be 25 times larger. Simply increasing โ„Ž by a factor of 5 wonโ€™t do it. Similarly, ifyou want to increase ๐‘‘ by a factor of 10, you need to increase โ„Ž by a factor of 100.Or said in a slightly different way, if you increase โ„Ž by a factor of 100, you increase๐‘‘ by a factor of only 10. Simple relations like these between ๐‘‘ and โ„Ž arenโ€™t obviousfrom looking at the correct (but not as simple) result in Eq. (6.21).

How close is the approximate answer in Eq. (6.23) to the exact answer inEq. (6.21) when ๐‘… = 6,400,000 and โ„Ž = 100? Plugging in the numbers gives (thefirst result here is just a repeat of Eq. (6.20), without the rounding)

๐‘‘exact =โˆš

2๐‘…โ„Ž + โ„Ž2 =โˆš

1,280,010,000 m2 = 35,777.23 m,

๐‘‘approx =โˆš

2๐‘…โ„Ž =โˆš

1,280,000,000 m2 = 35,777.09 m. (6.24)

We see that ourโˆš

2๐‘…โ„Ž approximation is a very good one. No one could possibly careabout an error of 0.14 m = 14 cm, when weโ€™re talking about distances of roughly36 km. Thereโ€™s truly no harm in ignoring the comparatively tiny โ„Ž2 term. Even atthe top of a tall mountain, the โ„Ž2 term will have no noticeable effect (when comparedwith the distance you can see).

When looking afar from peak,Remember this useful technique:In finding the distance,Ignore the existenceOf terms whose effect is quite weak.

Exercise 6.15 The height of the International Space Station is โ„Ž โ‰ˆ ๐‘…/16,which equals 250 miles, or 400 km. In terms of ๐‘…, find the exact andapproximate answers for ๐‘‘ in Eqs. (6.21) and (6.23). Then plug in ๐‘… =6,400 km to find the actual distances. By how much do they differ?

6.4. Distance to the horizon 271

What does Eq. (6.23) give for some other values of โ„Ž? If youโ€™re standing at theshore of the ocean, letโ€™s say that your eyes are at a height of โ„Ž = 2 m. We then have

๐‘‘ =โˆš

2๐‘…โ„Ž =โˆš

2(6,400,000 m) (2 m) = 5060 m โ‰ˆ 5 km โ‰ˆ 3 miles. (6.25)

On one hand, this might seem like a large distance, given that your eyes are only 2meters above the ground. But on the other hand, this distance is much smaller thanit would be if the earth were flat!

The values of ๐‘‘ โ‰ˆโˆš

2๐‘…โ„Ž for a few other values of โ„Ž are listed in Table 6.2.The โ„Žโ€™s are given in meters, and the ๐‘‘โ€™s are given in both kilometers and miles.Remember, to go from kilometers to miles, we multiply by 0.62. Are the variousvalues of ๐‘‘ larger or smaller than what you expected? (In some cases weโ€™ve keptmore significant figures than weโ€™re entitled to. Weโ€™ve done this so that you cancheck your calculations if you want to reproduce the numbers yourself. Weโ€™ve used6,400,000 m for ๐‘…, and 0.62 for the conversion from kilometers to miles, althoughthese are just rounded figures.)

Location โ„Ž (in m) ๐‘‘ (in km) ๐‘‘ (in miles)Standing ant 0.01 0.36 0.2 (1200 ft)Your eye near ground 0.1 1.1 0.7Person standing 2 5 3Somewhat tall building 100 36 22Burj Khalifa observatory 550 84 52Pikeโ€™s Peak, Colorado 4300 235 145Commercial airplane 10,000 358 222Space Station 400,000 2263 1403

Table 6.2: Distances to the horizon

Concerning the first entry in Table 6.2, the standing ant would need to be at theshore of very still water. Even the tiniest ripples (near where the tangent line inFig. 6.9 touches the earth) would ruin our perfect-sphere assumption for the earth.As โ„Ž gets larger, ripples (and eventually big waves) can be ignored.

Mt. Everest is about 8850 m tall, which is roughly the same as the 10,000 mcommercial-airplane entry in the table. So you can see about 200 miles from thetop of Mt. Everest. Or rather, you could see that far if there werenโ€™t other mountainsaround.

Depending on where the measurement is taken, the straight-line distance betweenthe east and west coasts of the US is about 2500 miles. Therefore, since the SpaceStation can see 1400 miles in either direction, for a total of 2800 miles, it can (justbarely) see both coasts at the same time.

272 Chapter 6. Pythagorean theorem

Hereโ€™s an easy formula to remember if you want to determine the distance ๐‘‘

associated with a given height โ„Ž. Let โ„Ž be ๐‘ meters. (So ๐‘ is just a pure numberwithout any units.) Then

๐‘‘ =โˆš

2๐‘…โ„Ž =โˆš

2(6,400,000 m)(๐‘ m) โ‰ˆ (3600 m)โˆš๐‘. (6.26)

So we can write ๐‘‘ as

๐‘‘ โ‰ˆ (3.6 km)โˆš๐‘ or (2.2 miles)

โˆš๐‘. (6.27)

So whatever the height โ„Ž is in meters, you simply need to take the square root ofthat and then multiply by either 3.6 km or 2.2 miles, depending on how you want toexpress your answer. But remember that in either case, ๐‘ is the number of meters.

Note that if we square both sides of Eq. (6.23), we obtain ๐‘‘2 = 2๐‘…โ„Ž. Dividingboth sides by 2๐‘… (and switching sides) then gives โ„Ž in terms of ๐‘‘:

โ„Ž =๐‘‘2

2๐‘…. (6.28)

This equation gives the answer to the question: If you want to see a given distance ๐‘‘,what does your height โ„Ž need to be? (This is the opposite of our original question offinding ๐‘‘ in terms of โ„Ž.) If you want to see ๐‘‘ = 160 km (100 miles), then Eq. (6.28)gives the required โ„Ž as (weโ€™ll work entirely with kilometers here, although you couldvery well use meters; there would just be some additional 0โ€™s in the numbers)

โ„Ž =(160 km)2

2(6,400 km) = 2 km = 2000 m. (6.29)

This translates to about 6,600 feet, which is a fairly tall mountain. This case liesbetween the Burj Khalifa and Pikeโ€™s Peak entries in Table 6.2.

Exercise 6.16 If you dig a straight tunnel from Boston to New York City(about 300 km apart), what is the depth โ„Ž of the tunnel at its deepest point?(Make a guess before solving the problem.) Hint: Draw a picture and lookfor a useful right triangle. Ignore the โ„Ž2 term you will encounter, as we didabove. Assume the earth is a perfect sphere.

6.5. Many examples and exercises 273

6.5 Many examples and exercises

Letโ€™s now do a number of examples and exercises. As always, you are encouragedto treat the examples as exercises and try them on your own first.

Example 6.2 In Example 5.4 we used the area of an octagon to produce anestimate of ๐œ‹. Letโ€™s obtain another estimate here, now using the perimeter.Weโ€™ll need to find the octagonโ€™s side length; the altitude we drew in Fig. 5.46is helpful for this.

Solution: Fig. 6.10 shows a 45โ—ฆ pie piece (just the triangle, without therounded end). Letting the radius be 1 as usual, the 45-45-90 triangle in theleft part of the pie piece has legs with lengths 1/

โˆš2 (from Section 5.4), as

shown.

1

s1

1 โˆ’

45

45

2/1 2/

1 2/

Figure 6.10

The important point to realize now is that the bottom side of the pie piece haslength 1, because itโ€™s also a radius. So a length 1 โˆ’ 1/

โˆš2 is left for the short

segment on the right side, as shown. The Pythagorean theorem applied to theright triangle in the right part of the pie piece then gives the octagonโ€™s sidelength ๐‘  as

๐‘ 2 =(1/โˆš

2)2 +

(1 โˆ’ 1/

โˆš2)2 = 1/2 +

(1 โˆ’ 2/

โˆš2 + 1/2

)= 2 โˆ’

โˆš2. (6.30)

Taking the square root of both sides yields ๐‘  =โˆš

2 โˆ’โˆš

2 โ‰ˆ 0.765, and thenmultiplying this by 8 to find the perimeter of the octagon gives ๐‘ƒoct โ‰ˆ 6.12.The ๐ถcirc > ๐‘ƒoct statement that the circumference of the circle is greater thanthe perimeter of the octagon is then 2๐œ‹ > 6.12, or equivalently ๐œ‹ > 3.06,after dividing by 2. This value is about 97% of the true ๐œ‹ โ‰ˆ 3.14 value, sothe approximation is a very good one.

274 Chapter 6. Pythagorean theorem

Example 6.3 Show that if the side lengths of a right triangle are equallyspaced (that is, if the hypotenuse exceeds the longer leg by the same amountthat the longer leg exceeds the shorter leg), then the sides are in the ratio of3 : 4 : 5.

Solution: Let ๐‘‘ be the difference between successive sides, and let the longerleg be ๐‘Ž. Then the side lengths are ๐‘Ž โˆ’ ๐‘‘, ๐‘Ž, and ๐‘Ž + ๐‘‘. So the Pythagoreantheorem gives

๐‘Ž2 + (๐‘Ž โˆ’ ๐‘‘)2 = (๐‘Ž + ๐‘‘)2 =โ‡’ ๐‘Ž2 = (๐‘Ž + ๐‘‘)2 โˆ’ (๐‘Ž โˆ’ ๐‘‘)2 (6.31)

=โ‡’ ๐‘Ž2 = (@@๐‘Ž2 + 2๐‘Ž๐‘‘ +๏ฟฝ๏ฟฝ๐‘‘2) โˆ’ (@@๐‘Ž2 โˆ’ 2๐‘Ž๐‘‘ +๏ฟฝ๏ฟฝ๐‘‘

2)=โ‡’ ๐‘Ž2 = 4๐‘Ž๐‘‘ =โ‡’ ๐‘Ž = 4๐‘‘ =โ‡’ ๐‘‘ = ๐‘Ž/4.

The second-to-last equation is obtained by dividing both sides of the previousone by ๐‘Ž, and then the last equation is obtained by further dividing by 4, andthen switching sides. (Or we could have simply divided by 4๐‘Ž in a singlestep.)

The ๐‘Ž โˆ’ ๐‘‘ leg of the triangle is therefore ๐‘Ž โˆ’ ๐‘‘ = ๐‘Ž โˆ’ ๐‘Ž/4 = 3๐‘Ž/4. And thehypotenuse is ๐‘Ž + ๐‘‘ = ๐‘Ž + ๐‘Ž/4 = 5๐‘Ž/4. So the three sides are 3๐‘Ž/4, ๐‘Ž, and5๐‘Ž/4. Scaling all of these up by a factor of 4 gives sides of 3๐‘Ž, 4๐‘Ž, and 5๐‘Ž.These are indeed in the ratio of 3 : 4 : 5.

Remarks:

1. Before doing any work on this problem, we already knew that a (3, 4, 5)right triangle has equally spaced lengths, since 5 โˆ’ 4 = 4 โˆ’ 3. What wedid in this solution was show that there are no other ratios (that is, no othershapes) that also have this property.

2. Letโ€™s be more precise about how we solved Eq. (6.31). When we arrivedat the ๐‘Ž2 = 4๐‘Ž๐‘‘ equation, what we technically should have done was toget everything on one side of the equation, and then factor. So subtracting4๐‘Ž๐‘‘ from both sides gives ๐‘Ž2 โˆ’ 4๐‘Ž๐‘‘ = 0, and then factoring this gives๐‘Ž(๐‘Ž โˆ’ 4๐‘‘) = 0. There are two ways the lefthand side can be zero. One isfor ๐‘Ž to be zero. However, although ๐‘Ž = 0 is a solution to our mathematicalequation, it isnโ€™t a solution to our problem, because the side lengths ๐‘Ž โˆ’ ๐‘‘,๐‘Ž, and ๐‘Ž+๐‘‘ are then โˆ’๐‘‘, 0, and ๐‘‘. And since side lengths must be positive,we therefore reject this solution, even though it does mathematically satisfy๐‘Ž2 + ๐‘2 = ๐‘2.

6.5. Many examples and exercises 275

The other solution is the one weโ€™re concerned with. The binomial ๐‘Ž โˆ’ 4๐‘‘is zero when ๐‘Ž = 4๐‘‘, or equivalently when ๐‘‘ = ๐‘Ž/4, as we found above.When we divided by ๐‘Ž above in Eq. (6.31), we were tacitly (and correctly)assuming that ๐‘Ž couldnโ€™t be zero.

3. In the above solution, we let the longer leg be ๐‘Ž. What if we instead let theshorter leg be ๐‘Ž? If ๐‘‘ is again the common spacing, the sides are now ๐‘Ž,๐‘Ž + ๐‘‘, and ๐‘Ž + 2๐‘‘. So the Pythagorean theorem gives

๐‘Ž2 + (๐‘Ž + ๐‘‘)2 = (๐‘Ž + 2๐‘‘)2

=โ‡’ ๐‘Ž2 + (๐‘Ž2 + 2๐‘Ž๐‘‘ + ๐‘‘2) = ๐‘Ž2 + 4๐‘Ž๐‘‘ + 4๐‘‘2. (6.32)

Getting all of the terms over on the righthand side by subtracting 2๐‘Ž2, 2๐‘Ž๐‘‘,and ๐‘‘2 from both sides gives

0 = 3๐‘‘2 + 2๐‘Ž๐‘‘ โˆ’ ๐‘Ž2. (6.33)

To solve this equation for ๐‘‘ in terms of ๐‘Ž, we can use the factoring methodwe learned in Section 4.2.3. After a little guessing and checking withFOIL, we find that Eq. (6.33) can be factored into

0 = (3๐‘‘ โˆ’ ๐‘Ž) (๐‘‘ + ๐‘Ž). (6.34)

The first binomial on the right side is zero when 3๐‘‘ = ๐‘Ž =โ‡’ ๐‘‘ = ๐‘Ž/3, andthe second is zero when ๐‘‘ = โˆ’๐‘Ž. This second root isnโ€™t allowed, becausethe side lengths ๐‘Ž, ๐‘Ž + ๐‘‘, and ๐‘Ž + 2๐‘‘ are then ๐‘Ž, 0, and โˆ’๐‘Ž. And as wenoted in the preceding remark, the side lengths must be positive.The ๐‘‘ = ๐‘Ž/3 solution is the one weโ€™re concerned with. The longer leg isthen ๐‘Ž + ๐‘‘ = ๐‘Ž + ๐‘Ž/3 = 4๐‘Ž/3, and the hypotenuse is ๐‘Ž + 2๐‘‘ = ๐‘Ž + 2 ยท ๐‘Ž/3 =5๐‘Ž/3. So the three sides are ๐‘Ž, 4๐‘Ž/3, and 5๐‘Ž/3. Scaling all of these upby a factor of 3 gives sides of 3๐‘Ž, 4๐‘Ž, and 5๐‘Ž, which are in the ratio of3 : 4 : 5. โ™ฃ

Example 6.4 In Fig. 6.11 circles with radii ๐‘Ž and ๐‘ lie on top of a line (thatis, they are tangent to it) and touch each other at a single point. What isthe distance ๐ต๐ด between the points of contact on the line? (Use the factthat a tangent line is perpendicular to the radius at the point of contact; seeExercise 6.24 below.)

Solution: The key is the shaded right triangle in the figure. The hypotenuseis the sum of the radii, so it equals ๐‘Ž + ๐‘, as shown. And the vertical leg is

276 Chapter 6. Pythagorean theorem

a

a

a โˆ’ b

b

B A

b

Figure 6.11

the difference of the radii, so it equals ๐‘Ž โˆ’ ๐‘. (This is how much higher onecenter is than the other.) The horizontal leg is the desired distance ๐ต๐ด, so thePythagorean theorem gives

(๐ต๐ด)2 + (๐‘Ž โˆ’ ๐‘)2 = (๐‘Ž + ๐‘)2

=โ‡’ (๐ต๐ด)2 = (๐‘Ž + ๐‘)2 โˆ’ (๐‘Ž โˆ’ ๐‘)2

= (๏ฟฝ๏ฟฝ๐‘Ž2 + 2๐‘Ž๐‘ +@@๐‘2) โˆ’ (๏ฟฝ๏ฟฝ๐‘Ž2 โˆ’ 2๐‘Ž๐‘ +@@๐‘

2)= 4๐‘Ž๐‘

=โ‡’ ๐ต๐ด =โˆš

4๐‘Ž๐‘ = 2โˆš๐‘Ž๐‘. (6.35)

Note that the algebraic steps here are the same as in Exercise 3.9.

In the special case where ๐‘Ž = ๐‘, Eq. (6.35) gives ๐ต๐ด = 2โˆš๐‘Ž2 = 2๐‘Ž. This

makes sense, because we have two equally sized circles sitting next to eachother, so ๐ต๐ด spans the sum of the two (equal) radii.

Remark: The quantityโˆš๐‘Ž๐‘ is called the geometric mean (GM) of ๐‘Ž and ๐‘.

Another type of mean is the arithmetic mean (AM), which is just the average(๐‘Ž+๐‘)/2. (Weโ€™ll talk about these means in Chapter 12.) Now, the hypotenuseof a right triangle, which is ๐‘Ž+ ๐‘ in the present setup, is always greater than orequal to each leg, in particular the horizontal leg which we just found equals2โˆš๐‘Ž๐‘ here. (They are equal if the other leg ๐‘Ž โˆ’ ๐‘ is zero, which happens if

๐‘Ž = ๐‘. The โ€œtriangleโ€ is then just a flat line, which admittedly isnโ€™t much of atriangle.) So we have

hypotenuse โ‰ฅ leg =โ‡’ ๐‘Ž + ๐‘ โ‰ฅ 2โˆš๐‘Ž๐‘ =โ‡’ ๐‘Ž + ๐‘

2โ‰ฅโˆš๐‘Ž๐‘, (6.36)

where we have divided both sides by 2. This result tells us that the arithmeticmean is always greater than or equal to the geometric mean (with equality

6.5. Many examples and exercises 277

occurring if ๐‘Ž = ๐‘). This statement is called the โ€œAM-GM inequality.โ€ Youcan test it for various values of ๐‘Ž and ๐‘. For example, if ๐‘Ž = 12 and ๐‘ = 3,then the AM is (12 + 3)/2 = 7.5, and the GM is

โˆš12 ยท 3 =

โˆš36 = 6. And 7.5

is indeed greater than 6.

With no mention of triangles, the above proof of the AM-GM inequalityessentially boils down to the following sequence of inequalities:

(๐‘Ž + ๐‘)2 โ‰ฅ (๐‘Ž + ๐‘)2 โˆ’ (๐‘Ž โˆ’ ๐‘)2 =โ‡’ (๐‘Ž + ๐‘)2 โ‰ฅ 4๐‘Ž๐‘ =โ‡’ ๐‘Ž + ๐‘

2โ‰ฅโˆš๐‘Ž๐‘.

(6.37)The first of these inequalities follows from the fact that (๐‘Ž โˆ’ ๐‘)2, being asquare, is always greater than or equal to zero (independent of which of ๐‘Ž or ๐‘is larger). And since weโ€™re subtracting it from (๐‘Ž + ๐‘)2 on the righthand side,the result must be less than or equal to (๐‘Ž + ๐‘)2. The second inequality comesfrom the cancelations in Eq. (6.35). And the third inequality is obtained bytaking the square root of both sides and then dividing both sides by 2.

In terms of the right triangle in Fig. 6.11, the lefthand side of the first inequalityin Eq. (6.37) is the square of the hypotenuse, and the righthand side is thesquare of the ๐ต๐ด leg (from the Pythagorean theorem usage in Eq. (6.35)). โ™ฃ

Example 6.5 Fig. 6.12 shows a right triangle with sides ๐‘Ž, ๐‘, and ๐‘. Theinscribed circle is drawn, and the contact points with the three sides dividethem into lengths ๐‘š, ๐‘›, and ๐‘Ÿ, as shown. (The lower-right lengths on sides ๐‘Žand ๐‘ are in fact equal to the radius ๐‘Ÿ , because of the square that arises fromthe fact that a radius is always perpendicular to a tangent; see Exercise 6.24below. Also, the two ๐‘š lengths shown are indeed equal, as are the two ๐‘›

lengths, because the two tangents drawn from a given point have the samelength; again see Exercise 6.24. Weโ€™ll just accept these facts here.)

br

r

r

r

r

a

c

n

n

m

m

Figure 6.12

278 Chapter 6. Pythagorean theorem

(a) Write down and simplify the statement of the Pythagorean theorem, whenwritten in terms of ๐‘š, ๐‘›, and ๐‘Ÿ.

(b) Find the area of the triangle in terms of ๐‘š, ๐‘›, and ๐‘Ÿ.

(c) Use your result from part (a) to rewrite the area, and show that it equals๐‘š๐‘› (the product of the lengths into which the hypotenuse is divided).

Solution:

(a) The side lengths are ๐‘š + ๐‘Ÿ, ๐‘› + ๐‘Ÿ, and ๐‘š + ๐‘›, so the Pythagorean theoremgives

(๐‘š + ๐‘Ÿ)2 + (๐‘› + ๐‘Ÿ)2 = (๐‘š + ๐‘›)2

=โ‡’ (๏ฟฝ๏ฟฝ๐‘š2 + 2๐‘š๐‘Ÿ + ๐‘Ÿ2) + (@@๐‘›2 + 2๐‘›๐‘Ÿ + ๐‘Ÿ2) = (๏ฟฝ๏ฟฝ๐‘š2 + 2๐‘š๐‘› +@@๐‘›2)

=โ‡’ 2๐‘š๐‘Ÿ + 2๐‘›๐‘Ÿ + 2๐‘Ÿ2 = 2๐‘š๐‘›

=โ‡’ ๐‘š๐‘Ÿ + ๐‘›๐‘Ÿ + ๐‘Ÿ2 = ๐‘š๐‘›. (6.38)

(b) Since the legs (the base and height of the triangle) have lengths ๐‘Ž = ๐‘š + ๐‘Ÿand ๐‘ = ๐‘› + ๐‘Ÿ, the area of the triangle is

๐ด =๐‘Ž๐‘

2=

(๐‘š + ๐‘Ÿ) (๐‘› + ๐‘Ÿ)2

=๐‘š๐‘› + ๐‘š๐‘Ÿ + ๐‘›๐‘Ÿ + ๐‘Ÿ2

2. (6.39)

(c) From Eq. (6.38), the sum of the last three terms in the numerator ofEq. (6.39) is ๐‘š๐‘›. Substituting this in, the area in Eq. (6.39) becomes

๐ด =๐‘š๐‘› + (๐‘š๐‘Ÿ + ๐‘›๐‘Ÿ + ๐‘Ÿ2)

2=๐‘š๐‘› + ๐‘š๐‘›

2=

AA2๐‘š๐‘›

AA2= ๐‘š๐‘›, (6.40)

as desired.

The above solution to this problem wasnโ€™t too long, but letโ€™s now spend sometime discussing various things further.

Remarks:

1. The simplicity of the above ๐‘š๐‘› result for the area suggests that there mightbe a clean geometric way of seeing why itโ€™s true, without needing to doany algebra. And indeed, if we consider the ๐‘š and ๐‘› segments on the legs(instead of on the hypotenuse) we can draw a suggestive figure. Fig. 6.13shows the idea.The goal is to show that the ๐‘š๐‘› area of the ๐‘š-by-๐‘› rectangle in the upper-left part of the figure equals the area of triangle ๐ด๐ต๐ถ, or equivalently

6.5. Many examples and exercises 279

r

rr

rr

r

r

n

AD

CB

n

m

m

Figure 6.13

triangle ๐ด๐ต๐ท. And this is indeed the case, because the ๐‘š-by-๐‘› rectanglecan be transformed into triangle ๐ด๐ต๐ท by moving the shaded triangles asshown. (The like-shaded right triangles are congruent, because they are (1)similar due to the common angle where they touch, and also the commonright angle, and hence the common third angle, and (2) they have the samesize due to the common ๐‘Ÿ leg.)

2. Since Eq. (6.38) tells us that ๐‘š๐‘› = ๐‘š๐‘Ÿ + ๐‘›๐‘Ÿ + ๐‘Ÿ2, the above ๐ด = ๐‘š๐‘› resultcan also be written as

๐ด = ๐‘š๐‘› = ๐‘š๐‘Ÿ + ๐‘›๐‘Ÿ + ๐‘Ÿ2 = (๐‘š + ๐‘› + ๐‘Ÿ)๐‘Ÿ = ๐‘†๐‘Ÿ, (6.41)

where ๐‘† is the โ€œsemiperimeterโ€ (half the perimeter). That is, ๐‘† = ๐‘ƒ/2 =(2๐‘š + 2๐‘› + 2๐‘Ÿ)/2 = ๐‘š + ๐‘› + ๐‘Ÿ . This ๐ด = ๐‘†๐‘Ÿ result actually holds for alltriangles, not just right triangles. To see why, consider Fig. 6.14, wheretriangle ๐ด๐ต๐ถ is divided into sub-triangles ๐ต๐ถ๐ท, ๐ถ๐ด๐ท, and ๐ด๐ต๐ท. Thesetriangles have bases ๐‘Ž, ๐‘, and ๐‘, and they all have the same altitude ๐‘Ÿ. Sothe sum of their areas (which is the area of triangle ๐ด๐ต๐ถ) is

๐‘Ž๐‘Ÿ

2+ ๐‘๐‘Ÿ

2+ ๐‘๐‘Ÿ

2=๐‘Ž + ๐‘ + ๐‘

2ยท ๐‘Ÿ = ๐‘ƒ

2๐‘Ÿ = ๐‘†๐‘Ÿ, (6.42)

as desired.

She found a new way to expressA triangleโ€™s ๐ด with success.The method prescribed:Take the circle inscribed,And then multiply ๐‘Ÿ by the ๐‘†.

Going one step further, this ๐ด = ๐‘†๐‘Ÿ result actually holds for all polygons(not just triangles) that have an inscribed circle, that is, one that tangen-tially touches all sides. (A randomly drawn polygon with four or more

280 Chapter 6. Pythagorean theorem

B C

D

b

a

c

rr

r

A

Figure 6.14

sides doesnโ€™t in general have this inscribed-circle property. Only specialpolygons do.) The triangle result in Eq. (6.42) is a special case of the moregeneral ๐ด = (๐‘ƒ/2)๐‘Ÿ = ๐‘†๐‘Ÿ result for polygons in Eq. (5.51) in the solutionto Exercise 5.24 (where ๐‘Ÿ was equal to 1).

3. Working backwards through the steps of this exercise, we can actuallyproduce another (an 8th!) proof of the Pythagorean theorem. Weโ€™ll startwith two expressions for the area (the standard ๐‘โ„Ž/2 one in Eq. (6.39), andthe ๐‘†๐‘Ÿ semiperimeter one in Eq. (6.42)) and equate them:

๐‘†๐‘Ÿ =๐‘โ„Ž

2=โ‡’ (๐‘š + ๐‘› + ๐‘Ÿ)๐‘Ÿ = (๐‘š + ๐‘Ÿ) (๐‘› + ๐‘Ÿ)

2. (6.43)

Weโ€™ll now proceed through a series of steps that will turn this equationinto the Pythagorean theorem. Expanding the products and multiplyingboth sides by 2 gives

2๐‘š๐‘Ÿ + 2๐‘›๐‘Ÿ + 2๐‘Ÿ2 = ๐‘š๐‘› + ๐‘š๐‘Ÿ + ๐‘›๐‘Ÿ + ๐‘Ÿ2. (6.44)

Subtracting ๐‘š๐‘Ÿ + ๐‘›๐‘Ÿ + ๐‘Ÿ2 from both sides then yields

๐‘š๐‘Ÿ + ๐‘›๐‘Ÿ + ๐‘Ÿ2 = ๐‘š๐‘›. (6.45)

Multiplying both sides by 2 and then adding ๐‘š2 + ๐‘›2 to both sides gives

2๐‘š๐‘Ÿ + 2๐‘›๐‘Ÿ + 2๐‘Ÿ2 + ๐‘š2 + ๐‘›2 = 2๐‘š๐‘› + ๐‘š2 + ๐‘›2. (6.46)

Finally, grouping the terms in a helpful manner yields

(๐‘š2 + 2๐‘š๐‘Ÿ + ๐‘Ÿ2) + (๐‘›2 + 2๐‘›๐‘Ÿ + ๐‘Ÿ2) = ๐‘š2 + 2๐‘š๐‘› + ๐‘›2

=โ‡’ (๐‘š + ๐‘Ÿ)2 + (๐‘› + ๐‘Ÿ)2 = (๐‘š + ๐‘›)2

=โ‡’ ๐‘Ž2 + ๐‘2 = ๐‘2, (6.47)

6.5. Many examples and exercises 281

which is the statement of the Pythagorean theorem, as desired.Admittedly, the above sequence of steps would be a bit out of the blue if wehadnโ€™t already solved the exercise in the โ€œforwardโ€ direction. But havingalready performed those steps, along with knowing what we were aimingfor (the second line in Eq. (6.47)), things were reasonably predictable.Once we produced Eq. (6.45), we just needed to proceed through Eq. (6.38)in reverse.In addition to the ๐‘โ„Ž/2 and ๐‘†๐‘Ÿ expressions for the area, there is alsothe ๐‘š๐‘› one, which is justified via Fig. 6.13. Equating any two of thesethree expressions will produce Eq. (6.45) (as you can quickly verify) andtherefore likewise produce the Pythagorean theorem. In other words, theequalities ๐‘โ„Ž/2 = ๐‘š๐‘› and ๐‘†๐‘Ÿ = ๐‘š๐‘› are the starting points of two moreproofs. If you want to count these as distinct new proofs, weโ€™re now up to10 of them! โ™ฃ

Exercise 6.17 A rectangular box has length ๐‘Ž, width ๐‘, and height ๐‘. Whatis the length of the diagonal between two opposite corners? (You will needto use the Pythagorean theorem twice.)

Exercise 6.18 A large cylindrical storage tank with diameter ๐‘‘ and heightโ„Ž has a spiral staircase, as shown in Fig. 6.15, which runs once around thecylinder as it climbs the height โ„Ž. (The slope of the staircase is uniform.)What is the length of the staircase? In the special case where ๐‘‘ = โ„Ž, what isthe length in terms of โ„Ž? Hint: A cylinder is a โ€œflatโ€ space, in the sense thatyou can make one out of a piece of paper without ripping the paper.

h

d

Figure 6.15

Exercise 6.19 A pendulum with length ๐‘… swings back and forth, moving inthe arc of a circle. When it is a distance ๐‘ฅ off to the side, as shown in Fig. 6.16,

282 Chapter 6. Pythagorean theorem

what is its height ๐‘ฆ above the lowest point, in terms of ๐‘ฅ and ๐‘…? Assume that๐‘ฆ is much smaller than ๐‘… (even though we havenโ€™t drawn it that way), andmake an appropriate approximation. Some helpful lines are drawn.

y

x

Rpath ofpendulum

pivot

Figure 6.16

Exercise 6.20 In addition to being the smallest Pythagorean triple, a 3-4-5right triangle shows up in a very simple geometric setup. Fig. 6.17 shows twoidentical large circles, both with radius 1, tangent to each other and to a line.A small circle is then drawn tangent to the two large circles and the line. Findthe radius ๐‘Ÿ of the small circle, and then show that the right triangle drawnhas a 3-4-5 shape.

rr

1 1

1

r

Figure 6.17

Exercise 6.21 In Exercise 5.22 we used the area of a dodecagon to producean estimate of ๐œ‹. Obtain another estimate here, now using the perimeter. Youwill need to find the dodecagonโ€™s side length; the altitude we drew in Fig. 5.53is helpful for this.

Exercise 6.22 The goal of this exercise is to generalize the procedure inExample 6.2 and Exercise 6.21.

(a) In Fig. 6.18, ๐ด๐ถ is a side of an ๐‘›-gon inscribed in a circle with radius1, and ๐ด๐ต and ๐ต๐ถ are sides of a 2๐‘›-gon. Assume that you somehow

6.5. Many examples and exercises 283

already know the ๐ด๐ถ = ๐‘Ž๐‘› side length of the ๐‘›-gon. Your task: Showthat the ๐ด๐ต = ๐ต๐ถ = ๐‘Ž2๐‘› side length of the 2๐‘›-gon is given in terms of๐‘Ž๐‘› by

๐‘Ž2๐‘› =

โˆš2 โˆ’ 2

โˆš1 โˆ’ ๐‘Ž2

๐‘›/4 . (6.48)

(You will need to find the ๐‘ฅ and ๐‘ฆ lengths shown.)

1

1

an/2

an/2

x

A

C

By

360 /n

360 /2n

a2n

Figure 6.18

(b) Using the known ๐‘Ž4 =โˆš

2 side length of a square inscribed in a circlewith radius 1, show that the above formula reproduces the result inExample 6.2 for the ๐‘Ž8 side of an octagon.Going one step further, what is ๐‘Ž16? And what is the resulting estimateof ๐œ‹? (Itโ€™s easier to work in terms of decimals here, so just plug yoursquare roots into a calculator.) You can keep going and obtain estimatesof ๐œ‹ by using a 32-gon and a 64-gon, etc., if you wish!

Exercise 6.23

(a) Show that if you inscribe a triangle in a circle, with a diameter being oneof the sides, then the triangle is a right triangle (with the diameter as thehypotenuse). Hint: Make use of the isosceles triangles in Fig. 6.19(a),after explaining why they are in fact isosceles.

(b) Now prove the โ€œreverseโ€ statement: Show that if you circumscribe acircle around a right triangle, then the hypotenuse is a diameter of thecircle. Hint: Your goal is to show that the midpoint of the hypotenuse is

284 Chapter 6. Pythagorean theorem

(b)

d

dd ?

(a)

Figure 6.19

the center of the circle, which is equivalent to showing that its distanceto the right-angled vertex (the long-dashed line drawn in Fig. 6.19(b)) isthe same as the (common) distance ๐‘‘ to the two other vertices. A helpfulvertical (short-dashed) line is drawn. Look for some similar triangles.

Exercise 6.24 A tangent is a line that touches a circle at exactly one point.That is, it just barely touches the circle. The word โ€œtangentโ€ can be used as anoun, as in the preceding sentence, and also as an adjective, as in โ€œThis lineis tangent to the circle.โ€

Weโ€™ve encountered tangents on a few occasions already. This exercise containstwo theorems about them. You might consider these theorems to be fairlyobvious, but itโ€™s still good to formally prove them.

(a) Prove that a tangent to a circle is perpendicular to the radius at thepoint of contact, as shown in Fig. 6.20(a). Hint: You can prove this bymaking use of the left/right symmetry in the picture. Or you can seewhat incorrect result the Pythagorean theorem would imply if the radiusand tangent werenโ€™t perpendicular.

(b) From a given point ๐‘ƒ outside a circle, draw the two tangent lines to thecircle. Prove that these two tangents have the same length, as shown inFig. 6.20(b).

Exercise 6.25 (This exercise is an extension of Example 6.4.) If we add athird circle to the setup in Fig. 6.11, as shown in Fig. 6.21, what is its radius๐‘Ÿ , in terms of the other two radii ๐‘Ž and ๐‘? Hint: Apply the result fromExample 6.4 multiple times, and use the fact that the segments along ๐ต๐ด mustadd up properly. Solving for ๐‘Ÿ in the equation you obtain will involve a fewalgebraic steps.

6.5. Many examples and exercises 285

A B

P

d d

(a) (b)

Figure 6.20

a

br

B C A

Figure 6.21

Exercise 6.26 (This exercise is very similar to Example 6.4.) Fig. 6.22 showsa circle with radius ๐‘ centered at ๐ต. From an arbitrary point ๐ด outside thecircle, the tangents (the dashed lines) are drawn; let their length be ๐‘Ž. A circlewith radius ๐‘Ž is then drawn, centered at ๐ด. Find the length ๐‘‘ of the commontangent to the two circles. Hint: You can quickly determine two sides of theshaded triangle.

bb

B

C

A

aa

d = ?

Figure 6.22

286 Chapter 6. Pythagorean theorem

6.6 Benefits of using letters

In Section 6.4 we solved the distance-to-horizon problem twice, first using numbers,and then using letters (plugging in the numbers only at the end). The logic behindthe solutions was the same, but they looked a bit different on paper. The techniqueof using letters instead of numbers is called solving a problem symbolically, whichbasically just means youโ€™re doing algebra (with letters) instead of arithmetic (withnumbers).

If youโ€™re solving a problem where the quantities are specified numerically, itis often advantageous to immediately change the numbers to letters (like replacing6,400 m with ๐‘…, and 100 m with โ„Ž in the horizon example). You can then solve theproblem in terms of the letters. After you obtain a symbolic answer, you can plug inthe actual numerical values to obtain a numerical answer. There are many benefitsof solving problems symbolically. And now that you have algebra at your fingertips,you should take advantage of these benefits. Letโ€™s list them out.

โ€ข It is quicker. Itโ€™s much easier to multiply an ๐‘… by an โ„Ž by writing themdown on a piece of paper next to each other, than it is to multiply theirnumerical values on a calculator. If solving a problem involves five or tensuch operations, the time would add up if you performed all the operations ona calculator.

โ€ข You are less likely to make a mistake. Numbers can get messy. Itโ€™s veryeasy to mistype an 8 for a 9 in a calculator, but youโ€™re probably not going tomiswrite a ๐‘˜ for an โ„Ž on a piece of paper. But even if you do, youโ€™ll quicklyrealize that it should be an โ„Ž. You certainly wonโ€™t just give up on the problemand deem it unsolvable because no one gave you the value of ๐‘˜!

โ€ข You can do the problem once and for all. If someone comes along andsays, oops, the value of โ„Ž is actually 90 m instead of 100 m, then you wonโ€™tneed to do the whole problem again. You can simply plug the new value ofโ„Ž into your symbolic answer. Thatโ€™s the beauty of working with letters. Asymbolic answer is valid for any value of the letter you might want to plugin (well, subject to any approximations, like the โ„Ž โ‰ช ๐‘… one in the horizonexample).

โ€ข You can see the general dependence of your answer on the variousparameters (letters). For example, you can see that the ๐‘‘ =

โˆš2๐‘…โ„Ž result

in Eq. (6.23) increases as either ๐‘… or โ„Ž increases. (For short, we say in this

6.6. Benefits of using letters 287

case that โ€œ๐‘‘ grows with ๐‘… and โ„Ž.โ€) Furthermore, you can see how ๐‘‘ growswith ๐‘… and โ„Ž: It grows in a square-root manner for both. So if you increase โ„Ž

by a factor of 100, you can see only 10 times as far. Equivalently, the square(instead of square root) behavior in the โ„Ž = ๐‘‘2/2๐‘… result in Eq. (6.28) tells usthat if we increase the distance ๐‘‘ by a factor of 10, then we need to increase โ„Ž

by a factor of 100. There is much more information contained in the symbolicanswer in Eq. (6.23) than in the numerical answer in Eq. (6.20).

As a bonus, symbolic answers nearly always look nice and pretty. Even incases like in Eq. (6.21) where the symbolic answer isnโ€™t super pretty, there isstill a huge amount of information. Under the โ„Ž โ‰ช ๐‘… approximation, youcan say that the โ„Ž2 term is very small compared with the 2๐‘…โ„Ž term, whichmeans that you can ignore it. This leaves you with the result in Eq. (6.23),which is in fact super pretty.

โ€ข You can check special/extreme cases. (This is a long bullet point, sinceitโ€™s so important.) This benefit goes hand-in-hand with the previous โ€œgeneraldependenceโ€ advantage. Since symbolic answers allow you to see the depen-dence on the various letters, you can easily determine what your answer is(or at least how it behaves) in various special or extreme cases. For example,perhaps you can determine what your answer is when a particular letter equalszero. Or when it is very large. Or when two letters are equal to each other.And so on.

It is often the case that your intuition gives you information about what theanswer should be in special/extreme cases, even if you donโ€™t have any intuitionabout general values of the letters. You should take advantage of this. Forexample, I have no clue how far I can see to the horizon from a height of,say, 500 meters. But I do know for sure that I can see zero distance fromzero height. (Itโ€™s up to you whether you want to call this โ€œintuitionโ€ or just anobvious fact.) And indeed, the ๐‘‘ =

โˆš2๐‘…โ„Ž result in Eq. (6.23) correctly equals

zero when โ„Ž = 0. If you accidentally replaced the โ„Ž here with ๐‘… and endedup with an answer of

โˆš2๐‘…2, or if you simply forgot the โ„Ž and ended up withโˆš

2๐‘…, then these answers donโ€™t equal zero when โ„Ž = 0. So youโ€™d know thatyou needed to go back and check over your work. Likewise if you accidentallywrote the result in Eq. (6.21) as, say,

โˆš2๐‘…โ„Ž + ๐‘…2.

You donโ€™t need to be a magician,Or cough up a hefty tuition.When you check an extreme,Thereโ€™s a nice simple scheme:Test your answer against intuition!

288 Chapter 6. Pythagorean theorem

In the other extreme, where โ„Ž gets very large, the ๐‘‘ =โˆš

2๐‘…โ„Ž + โ„Ž2 result inEq. (6.21) also gets very large, which makes sense; ๐‘‘ correctly grows as โ„Ž

grows. (For large โ„Ž, theโˆš

2๐‘…โ„Ž result in Eq. (6.23) isnโ€™t valid, since it wasderived under the assumption that โ„Ž is much smaller than ๐‘…. So we need touse the

โˆš2๐‘…โ„Ž + โ„Ž2 expression here.) In particular, if โ„Ž is much larger than ๐‘…

(imagine that youโ€™re on the moon, looking at the earth), then the 2๐‘…โ„Ž termis small compared with the โ„Ž2 term (it is smaller by the factor 2๐‘…/โ„Ž). So toa reasonable approximation, we can ignore the 2๐‘…โ„Ž in

โˆš2๐‘…โ„Ž + โ„Ž2, in which

case weโ€™re left with ๐‘‘ โ‰ˆโˆšโ„Ž2 = โ„Ž. This makes sense; from the moon, the

distance ๐‘‘ to the earthโ€™s horizon is approximately equal to the distance โ„Ž tothe nearest point on the earth; see Fig. 6.23.

R

R h

d

you

Figure 6.23

A better intuitive approximation for ๐‘‘ is โ„Ž + ๐‘…, due to the extra distance ofroughly ๐‘… to reach the horizon, as opposed to just the nearest point on theearth. Equivalently, the long leg ๐‘‘ in Fig. 6.23 is approximately equal tothe hypotenuse โ„Ž + ๐‘… since the right triangle is very thin. (However, weโ€™reassuming ๐‘… is much smaller than โ„Ž, so the distinction between โ„Ž and โ„Ž+๐‘… isnโ€™ttoo important.) If you made a mistake and obtained a ๐‘‘ of, say,

โˆš2๐‘…โ„Ž + 2โ„Ž2,

then ignoring the 2๐‘…โ„Ž term would yield an approximate answer ofโˆš

2โ„Ž. Whenโ„Ž is large, this answer is much larger than either of our approximate intuitiveanswers (โ„Ž or โ„Ž + ๐‘…). It therefore canโ€™t be correct. So youโ€™d know to go backand check over your work.

As another example, consider the 2๐ฟ๐‘Ž + 2๐‘Š๐‘Ž + 4๐‘Ž2 result in Eq. (5.5) inExample 5.1. In the special case where ๐‘Ž = 0, the frame has no width, so thearea clearly has to be zero. And the above answer is indeed zero when ๐‘Ž = 0.If you made a mistake and accidentally replaced the first ๐‘Ž with an ๐ฟ, yielding2๐ฟ2 + 2๐‘Š๐‘Ž + 4๐‘Ž2, then youโ€™d know this couldnโ€™t be correct, because it equals2๐ฟ2, instead of zero, when ๐‘Ž = 0.

Likewise for the 2๐œ‹๐‘Ÿ๐‘Ž + ๐œ‹๐‘Ž2 result in Eq. (5.17) for the area of a ring betweentwo circles. The answer must equal zero when the thickness ๐‘Ž of the ringis zero, and the above expression correctly has this property. Furthermore,when ๐‘Ž is very small, but not exactly zero, we can ignore the very small ๐‘Ž2

term, in which case the expression reduces to 2๐œ‹๐‘Ÿ๐‘Ž. This is consistent with

6.6. Benefits of using letters 289

the fact that the area of the ring is (when ๐‘Ž is very small) essentially equalto the circumference 2๐œ‹๐‘Ÿ times the thickness ๐‘Ž, as we saw in Eq. (5.19). Ifyou made a mistake and dropped the 2 and obtained an answer of ๐œ‹๐‘Ÿ๐‘Ž + ๐œ‹๐‘Ž2,then although the ๐‘Ž = 0 special case correctly yields zero, the ๐‘Ž-small-but-not-exactly-zero case doesnโ€™t yield 2๐œ‹๐‘Ÿ times ๐‘Ž. So youโ€™d know to go backand check over your work. No matter how many special cases you check thatare correct, if you obtain even just one that isnโ€™t correct, then you know thatyour answer must be wrong.

Another example of checking a special case is the ๐‘Ž8 โˆ’ ๐‘8 = (๐‘Ž4 + ๐‘4) (๐‘Ž2 +๐‘2)(๐‘Ž + ๐‘) (๐‘Ž โˆ’ ๐‘) factoring result in Eq. (4.38) in Example 4.6. In the specialcase where ๐‘Ž = ๐‘, the lefthand side is zero. And the righthand side is correctlyalso zero. (Likewise for the ๐‘Ž = โˆ’๐‘ special case; both sides are zero.) If youforgot the ๐‘Ž โˆ’ ๐‘ factor on the right, then the righthand side wouldnโ€™t be zero,so youโ€™d know you made a mistake in your factoring. As another example, inthe solution to Example 6.4 we noted that the ๐ต๐ด = 2

โˆš๐‘Ž๐‘ result in Eq. (6.35)

correctly reduces to 2๐‘Ž in the special case where ๐‘Ž = ๐‘.

Bottom line: When you arrive at an answer after solving a problem, youshould always look for special/extreme cases to check. And you should dothis not because Iโ€™m telling you to(!), but rather because it will either (a) giveyou the definite information that your answer is incorrect (in which case younow know you need to go fix it), or (b) allow you to feel a little more confidentabout your answer if youโ€™ve checked a number of special/extreme cases andthey all agree with what you know must be true. Such is the case with the sumformulas in Exercises 4.16, 4.17, and 4.18. After checking those formulasfor a number of small values of ๐‘›, you will certainly be more confident thattheyโ€™re actually correct.

Of course, checking special/extreme cases will never tell you that your answeris definitely correct. Itโ€™s quite possible that youโ€™ve produced an incorrectanswer that just happens by luck to give the correct answer for a number ofspecial cases. However, as weโ€™ve noted, looking at a special/extreme casemight very well tell you that your answer is definitely incorrect. If pluggingin a special value for a letter gives an answer that doesnโ€™t agree with youintuition, then (assuming that your intuition is correct) you have obtained theirrefutable information that your answer is wrong. This seemingly dispiritinginformation is actually a good thing, because as mentioned above, at least younow know that you should go back and check over your work. This outcomecertainly beats pressing onward in blissful ignorance, thinking that you havethe correct answer, when in fact you donโ€™t!

290 Chapter 6. Pythagorean theorem

โ€ข You can check units. In addition to checking special/extreme cases, sym-bolic answers also allow you to easily check units. In the ๐‘‘ =

โˆš2๐‘…โ„Ž result

in Eq. (6.23), both ๐‘… and โ„Ž have units of meters (or feet, or whatever unit oflength youโ€™re working with). So the units of the ๐‘‘ are

โˆšm ยท m =

โˆšm2 = m,

which is correct. (In determining the units, we can ignore the numerical factor2, since it doesnโ€™t have any units.)

If you made a mistake and obtained an answer ofโˆš

2๐‘…/โ„Ž, with the โ„Ž inthe denominator, then youโ€™d know it had to be wrong, because the units areโˆš

m/m =โˆš

1 = 1, where the 1 here simply means thatโˆš

2๐‘…/โ„Ž doesnโ€™t haveany units. This is incorrect, since ๐‘‘ must have units of meters. Similarly, ifyou accidentally dropped the ๐‘… and obtained an answer of

โˆš2โ„Ž, this has units

ofโˆš

m, which is incorrect.

Of course, the units will also work out (assuming you donโ€™t make a mistake)if you solve a problem in terms of numbers instead of letters, as we saw inEqs. (6.18)โ€“(6.20). You therefore can (and should) also check the units ofyour answer when working with numbers. But again, solving a problem interms of numbers instead of letters can often be a pain.

In summary, there are many significant benefits of using letters instead of num-bers. And now that youโ€™ve had lots of practice with algebra, youโ€™re able to workwith letters at will. So take advantage of all of their wonderful benefits โ€“ theyโ€™llmake your life much more pleasant!

They strove to be mighty trend setters,And be free from numerical fetters.Their motto on numbers?โ€œReject what encumbers!And bask in the glory of letters!โ€

6.7. Exercise solutions 291

6.7 Exercise solutions

1. Plugging the ๐‘Ž and ๐‘ expressions from Eq. (6.8) into Eq. (6.1) gives

๐‘Ž2 + ๐‘2 = (๐‘š2 โˆ’ ๐‘›2) + (2๐‘š๐‘›)2

= (๐‘š4 โˆ’ 2๐‘š2๐‘›2 + ๐‘›4) + 4๐‘š2๐‘›2

= ๐‘š4 + 2๐‘š2๐‘›2 + ๐‘›4

= (๐‘š2 + ๐‘›2)2

= ๐‘2, (6.49)

as desired.

2. Since ๐‘2 โˆ’ ๐‘2 equals (๐‘ + ๐‘)(๐‘ โˆ’ ๐‘), our goal is to show that this product equals๐‘Ž2. Plugging in the expressions for ๐‘ and ๐‘ from Eq. (6.8) gives

๐‘2 โˆ’ ๐‘2 = (๐‘ + ๐‘)(๐‘ โˆ’ ๐‘) (6.50)

=((๐‘š2 + ๐‘›2) + 2๐‘š๐‘›

) ((๐‘š2 + ๐‘›2) โˆ’ 2๐‘š๐‘›

)= (๐‘š + ๐‘›)2(๐‘š โˆ’ ๐‘›)2 =

((๐‘š + ๐‘›) (๐‘š โˆ’ ๐‘›)

)2 = (๐‘š2 โˆ’ ๐‘›2)2 = ๐‘Ž2,

as desired. In the last line, we used the difference-of-squares result in reverse.

3. Weโ€™ll just check a few of the triples here. The following relations are all true:

72 + 242 = 252 โ‡โ‡’ 49 + 576 = 625,212 + 202 = 292 โ‡โ‡’ 441 + 400 = 841,92 + 402 = 412 โ‡โ‡’ 81 + 1600 = 1681. (6.51)

4. From the Pythagorean theorem, the diagonal of 11.3-by-7 rectangle isโˆš11.32 + 72 =

โˆš176.7 = 13.3. (6.52)

And the diagonal of a 14.4-by-9.0 rectangle isโˆš14.42 + 92 =

โˆš288.4 = 17. (6.53)

For the first of these, people usually just call it a 13-inch screen, althoughtechnically itโ€™s 13.3.

There are many (an infinite number of) other shapes of rectangles, with differentwidth-to-height ratios (called the โ€œaspect ratioโ€), that also have diagonals of 13.3and 17. However, computers generally stick to a few common aspect ratios, like16 : 10 (equivalently 8 : 5), 16 : 9, and 4 : 3.

292 Chapter 6. Pythagorean theorem

5. If you walk along two sides, the distance (in yards) is simply 100+53.33 = 153.33.From the Pythagorean theorem, the length of the diagonal isโˆš

1002 + 53.332 =โˆš

12,844 = 113.33. (6.54)

So you save 153.33 โˆ’ 113.33 = 40 yards by walking diagonally. The diagonalisnโ€™t that much longer than the long side (113 vs. 100), so you save almost asmuch as the short side (40 vs. 53).

Interestingly, if youโ€™ve ever wondered how big an acre is, itโ€™s a little less thana football field. This follows from the fact that an acre is defined to be 43,560square feet (1/640 of a square mile, as you can verify, since 1 mile = 5,280 feet),and a football field (300 feet by 160 feet) is 48,000 square feet.

6. If ๐‘› = 1, the expressions in Eq. (6.8) become

๐‘Ž = ๐‘š2 โˆ’ 1, ๐‘ = 2๐‘š, ๐‘ = ๐‘š2 + 1. (6.55)

Since ๐‘/2 is simply๐‘š, we see that ๐‘Ž and ๐‘ do indeed take the form of (๐‘/2)2ยฑ1 =๐‘š2 ยฑ 1.

7. Starting with ๐‘Ž = 2๐‘˜ โˆ’ 1 and following the given recipe, we first square ๐‘Ž toobtain

๐‘Ž2 = (2๐‘˜ โˆ’ 1)2 = 4๐‘˜2 โˆ’ 4๐‘˜ + 1. (6.56)

We then add or subtract 1 to obtain 4๐‘˜2 โˆ’ 4๐‘˜ + 2 and 4๐‘˜2 โˆ’ 4๐‘˜ . Finally, we divideby 2 and label the results as ๐‘ and ๐‘:

๐‘ = 2๐‘˜2 โˆ’ 2๐‘˜ + 1, ๐‘ = 2๐‘˜2 โˆ’ 2๐‘˜. (6.57)

Our goal is to show that (๐‘Ž, ๐‘, ๐‘) is a Pythagorean triple, that is, that ๐‘Ž2+๐‘2 = ๐‘2.The sum ๐‘Ž2 + ๐‘2 equals

๐‘Ž2 + ๐‘2 = (2๐‘˜ โˆ’ 1)2 + (2๐‘˜2 โˆ’ 2๐‘˜)2

= (4๐‘˜2 โˆ’ 4๐‘˜ + 1) + (4๐‘˜4 โˆ’ 8๐‘˜3 + 4๐‘˜2)= 4๐‘˜4 โˆ’ 8๐‘˜3 + 8๐‘˜2 โˆ’ 4๐‘˜ + 1. (6.58)

We want to show that this equals ๐‘2, which is obtained by squaring the trinomialfor ๐‘ in Eq. (6.57). Using the trinomial result from Example 3.6, we obtain

(2๐‘˜2 โˆ’ 2๐‘˜ + 1)2 =((2๐‘˜2)2 + (โˆ’2๐‘˜)2 + 12

)+ 2

((2๐‘˜2) (โˆ’2๐‘˜) + (2๐‘˜2) (1) + (โˆ’2๐‘˜)(1)

)=

(4๐‘˜4 + 4๐‘˜2 + 1

)+

(โˆ’ 8๐‘˜3 + 4๐‘˜2 โˆ’ 4๐‘˜

)= 4๐‘˜4 โˆ’ 8๐‘˜3 + 8๐‘˜2 โˆ’ 4๐‘˜ + 1, (6.59)

6.7. Exercise solutions 293

in agreement with the ๐‘Ž2 + ๐‘2 sum in Eq. (6.58), as desired.

This calculation was a bit long, and there were many places where we could havemade a mistake. But we were careful, and it all worked out. The reward for theeffort was two long expressions that were exactly equal to each other. Perhapsthe most likely error in the calculation is forgetting to distribute the 2 into allthree terms in the parentheses in the second line of Eq. (6.59).

The purpose of this exercise was to get some algebra practice. If the goal wereinstead to show as quickly as possible that ๐‘Ž2+๐‘2 = ๐‘2 (once ๐‘ and ๐‘ are obtainedin Eq. (6.57)), then the best route would be to write this as ๐‘Ž2 = ๐‘2 โˆ’ ๐‘2, andthen use the difference-of-squares result. This method is much quicker, as youcan verify!

Note that our starting ๐‘Ž = 2๐‘˜ โˆ’ 1 expression, along with the expressions for ๐‘and ๐‘ in Eq. (6.57), are the same as the expressions for ๐‘Ž, ๐‘, and ๐‘ in Eq. (6.10)(which came from Eq. (6.8)), with ๐‘š replaced by ๐‘˜ . So we already knew fromour original proof in Exercise 6.1 that the ๐‘Ž2 + ๐‘2 = ๐‘2 relation holds. But again,the purpose of this exercise was to verify that ๐‘Ž2 + ๐‘2 = ๐‘2 holds by doing somealgebra.

8. The overall square has side length ๐‘Ž + ๐‘, so its area is (๐‘Ž + ๐‘)2. The area of thesmaller square is ๐‘2. And the area of each of the four triangles is ๐‘Ž๐‘/2 (sinceeach one has a base of ๐‘Ž and a height of ๐‘, or vice versa). So the statement thatthe overall area equals the sum of the areas of the sub-regions is

(๐‘Ž + ๐‘)2 = 4 ยท ๐‘Ž๐‘2

+ ๐‘2

=โ‡’ ๐‘Ž2 +๏ฟฝ๏ฟฝ๏ฟฝ2๐‘Ž๐‘ + ๐‘2 =๏ฟฝ๏ฟฝ๏ฟฝ2๐‘Ž๐‘ + ๐‘2

=โ‡’ ๐‘Ž2 + ๐‘2 = ๐‘2, (6.60)

as desired. The third line is obtained by canceling the 2๐‘Ž๐‘ terms on each sideof the second line (or more precisely, by subtracting 2๐‘Ž๐‘ from both sides of thesecond line, as explained in the second terminology bullet point on page 163).

9. The area of the overall square is ๐‘2, and the area of the smaller square is (๐‘โˆ’ ๐‘Ž)2.The area of each of the four triangles is ๐‘Ž๐‘/2 (since each one has a base of ๐‘Ž anda height of ๐‘, or vice versa), So the statement that the overall area equals the sumof the areas of the sub-regions is

๐‘2 = 4 ยท ๐‘Ž๐‘2

+ (๐‘ โˆ’ ๐‘Ž)2

=๏ฟฝ๏ฟฝ๏ฟฝ2๐‘Ž๐‘ + (๐‘2 โˆ’๏ฟฝ๏ฟฝ๏ฟฝ2๐‘Ž๐‘ + ๐‘Ž2)= ๐‘2 + ๐‘Ž2, (6.61)

as desired.

294 Chapter 6. Pythagorean theorem

10. Fig. 6.24 shows the rearrangement. The white squares that are formed have sidelengths ๐‘Ž and ๐‘, because those are the legs of the four shaded rectangles in theoriginal figure. So the area of the white region is indeed ๐‘Ž2 + ๐‘2.

Note that we donโ€™t even need to know that the area of a triangle is ๐‘โ„Ž/2 here, aswe did in the first two proofs above. All we need to know is that the area of asquare is the side length squared.

bb

b

c b2

a2

c2

ba

a

a

a

Figure 6.24

11. We just need to move the two dark-shaded triangles in Fig. 6.25 as indicated. (Itdoesnโ€™t matter which one goes where, since theyโ€™re identical.) The total shadedarea (light and dark) is the same in the two figures. And since this area is ๐‘Ž2 + ๐‘2

in the left figure and ๐‘2 in the right, it follows that ๐‘Ž2 + ๐‘2 = ๐‘2.

a

a

a

b

b

b c

ca

a

a

b

b

b c

c

Figure 6.25

This proof and the preceding one show that sometimes a proof doesnโ€™t requireany words (even though we did use some). A simple picture by itself can do thetrick!

6.7. Exercise solutions 295

The proof she gave somehow succeeded(a bit odd, given how it proceeded).But the picture she drewSoon convinced us itโ€™s trueThat in some cases words are not needed!

12. For convenience in Fig. 6.26, let โˆ ๐ด be labeled as๐›ผ, as shown. Then โˆ ๐ต = 90โ—ฆโˆ’๐›ผbecause โˆ ๐ด๐ถ๐ต = 90โ—ฆ, and the three angles in triangle ๐ด๐ต๐ถ must add up to 180โ—ฆ.

a

A

B

C

Da2/c

ab/cb2/c

ฮฑ

ฮฑ

b

90 โˆ’ ฮฑ

c90 โˆ’ ฮฑ

Figure 6.26

Similarly, in right triangle ๐ด๐ถ๐ท, we have โˆ ๐ด๐ถ๐ท = 90โ—ฆโˆ’๐›ผ because โˆ ๐ด๐ท๐ถ = 90โ—ฆ,and the three angles in triangle ๐ด๐ถ๐ท must add up to 180โ—ฆ.

Finally, due to the original right angle at ๐ถ, we have

โˆ ๐ท๐ถ๐ต = 90โ—ฆ โˆ’ โˆ ๐ท๐ถ๐ด = 90โ—ฆ โˆ’ (90โ—ฆ โˆ’ ๐›ผ) = ๐›ผ. (6.62)

You can also deduce this ๐›ผ angle by demanding that the angles in triangle ๐ถ๐ต๐ท

add up to 180โ—ฆ.

We therefore see that all three of the triangles ๐ด๐ต๐ถ, ๐ด๐ถ๐ท, and๐ถ๐ต๐ท have anglesof 90โ—ฆ, ๐›ผ, and 90โ—ฆ โˆ’ ๐›ผ. Hence they are all similar, as we wanted to show.

We can now use the similarity of the triangles to write down some useful ratios.In the overall triangle ๐ด๐ต๐ถ, the short leg ๐‘Ž is ๐‘Ž/๐‘ times the hypotenuse ๐‘, andthe long leg ๐‘ is ๐‘/๐‘ times the hypotenuse ๐‘. Since the other two smaller righttriangles are similar to the overall one, they must have these same ratios. Thatis, in each triangle, the short leg is ๐‘Ž/๐‘ times the hypotenuse, and the long leg is๐‘/๐‘ times the hypotenuse.

So in the smallest triangle, the short leg ๐ต๐ท is ๐‘Ž/๐‘ times the hypotenuse, whichis ๐ถ๐ต = ๐‘Ž. So ๐ต๐ท = (๐‘Ž/๐‘)๐‘Ž = ๐‘Ž2/๐‘, as shown.

And in the medium triangle, the long leg ๐ด๐ท is ๐‘/๐‘ times the hypotenuse, whichis ๐ด๐ถ = ๐‘. So ๐ด๐ท = (๐‘/๐‘)๐‘ = ๐‘2/๐‘, as shown.

296 Chapter 6. Pythagorean theorem

We can now use the fact that ๐ต๐ท + ๐ด๐ท equals the hypotenuse ๐‘ of the overalltriangle. This gives

๐ต๐ท + ๐ด๐ท = ๐‘ =โ‡’ ๐‘Ž2

๐‘+ ๐‘2

๐‘= ๐‘ =โ‡’ ๐‘Ž2 + ๐‘2 = ๐‘2, (6.63)

where this last equation is obtained from the middle one by multiplying bothsides by ๐‘.

Remark: We can also find the length of the altitude, ๐ถ๐ท, in various differentways. In the smallest triangle, the long leg ๐ถ๐ท is ๐‘/๐‘ times the hypotenuse,which is ๐ถ๐ต = ๐‘Ž. So ๐ถ๐ท = (๐‘/๐‘)๐‘Ž = ๐‘Ž๐‘/๐‘, as shown.

Alternatively, in the medium triangle, the short leg ๐ถ๐ท is ๐‘Ž/๐‘ times the hy-potenuse, which is ๐ด๐ถ = ๐‘. So ๐ถ๐ท = (๐‘Ž/๐‘)๐‘ = ๐‘Ž๐‘/๐‘.

Alternatively again, we can find๐ถ๐ท by writing down two valid ๐‘โ„Ž/2 expressionsfor the area of the overall triangle. We can consider ๐‘ to be the base and ๐‘Ž tobe the height. Or we can consider ๐‘ to be the base and ๐ถ๐ท to be the height.Equating the two resulting expressions for the area gives

๐‘๐‘Ž

2=๐‘(๐ถ๐ท)

2=โ‡’ ๐‘Ž๐‘

๐‘= ๐ถ๐ท , (6.64)

where the second equation is obtained from the first by multiplying both sidesby 2/๐‘. โ™ฃ

13. Let the three triangles (overall, medium, and smallest) be labeled ๐‘‡1, ๐‘‡2, and ๐‘‡3,respectively. The hypotenuse of ๐‘‡1 is ๐‘, and the hypotenuse of ๐‘‡2 is ๐‘. So ๐‘‡2 isobtained by scaling down ๐‘‡1 by the factor ๐‘“ = ๐‘/๐‘. The results from Section 5.5then tell us that the areas are related by ๐ด2 = ๐‘“ 2๐ด1 =โ‡’ ๐ด2 = (๐‘/๐‘)2๐ด1.

Similarly, the hypotenuse of ๐‘‡3 is ๐‘Ž, so ๐‘‡3 is obtained by scaling down ๐‘‡1 bythe factor ๐‘“ = ๐‘Ž/๐‘. The areas are therefore related by ๐ด3 = ๐‘“ 2๐ด1 =โ‡’ ๐ด3 =(๐‘Ž/๐‘)2๐ด1.

The ๐ด2 + ๐ด3 = ๐ด1 relation for areas then becomes

๐‘2

๐‘2 ๐ด1 +๐‘Ž2

๐‘2 ๐ด1 = ๐ด1 =โ‡’ ๐‘2

๐‘2 + ๐‘Ž2

๐‘2 = 1 =โ‡’ ๐‘2 + ๐‘Ž2 = ๐‘2, (6.65)

as desired. The last equation is obtained by dividing both sides of the firstequation by ๐ด1, and then multiplying both sides of the second equation by ๐‘2.(Or you can just multiply by ๐‘2/๐ด1 in one step.)

6.7. Exercise solutions 297

14. The dark-shaded triangles ๐ด1๐ต๐ด and ๐ถ๐ต๐ถ1 in Fig. 6.7 both have sides withlengths ๐‘Ž and ๐‘. So if we can show that the angles โˆ ๐ด1๐ต๐ด and โˆ ๐ถ๐ต๐ถ1 betweenthese common sides are equal, then by the SAS postulate weโ€™ll know that the twotriangles are congruent. These angles are indeed equal because

โˆ ๐ด1๐ต๐ด = โˆ ๐ด1๐ต๐ถ + โˆ ๐ถ๐ต๐ด = 90โ—ฆ + โˆ ๐ถ๐ต๐ด,

and โˆ ๐ถ๐ต๐ถ1 = โˆ ๐ด๐ต๐ถ1 + โˆ ๐ถ๐ต๐ด = 90โ—ฆ + โˆ ๐ถ๐ต๐ด. (6.66)

So both angles are 90โ—ฆ plus โˆ ๐ถ๐ต๐ด, and hence are equal. Triangles ๐ด1๐ต๐ด and๐ถ๐ต๐ถ1 are therefore congruent by the SAS postulate. So they have the same area.

Note: Another (quick) way of seeing why triangles ๐ด1๐ต๐ด and ๐ถ๐ต๐ถ1 are con-gruent is to imagine rotating triangle ๐ถ๐ต๐ถ1 clockwise by 90โ—ฆ around point ๐ต. Itwill turn into triangle ๐ด1๐ต๐ด.

All of the above reasoning holds with the light-shaded triangles too, so they arealso congruent and hence have the same area.

We now claim that the area of triangle ๐ด1๐ต๐ด is half the area of the square withside ๐‘Ž. This is true because in the standard (1/2)๐‘โ„Ž expression for the area of atriangle, we can pick the base to be the ๐ด1๐ต = ๐‘Ž side, in which case the altitudefrom ๐ด to the (extension of the) ๐ด1๐ต side has length ๐‘Ž, as shown in Fig. 6.27(a).So the area of triangle ๐ด1๐ต๐ด is (1/2)๐‘โ„Ž = (1/2)๐‘Ž ยท ๐‘Ž = ๐‘Ž2/2, which is indeedhalf the area of the square with side ๐‘Ž.

a

C1

C

B

c

x

ah = a

h = x

a

A2

A1 R1

C

(a) (b)

B

(height)

(height)

(base)

(base)

A

c

Figure 6.27

We also claim that the area of triangle๐ถ๐ต๐ถ1 is half the area of the ๐‘…1 rectangularpart of the square with side ๐‘ that is above/left of the dashed line. This is true

298 Chapter 6. Pythagorean theorem

because in the standard (1/2)๐‘โ„Ž expression for the area of a triangle, we canpick the base to be the ๐ต๐ถ1 = ๐‘ side, in which case the altitude from ๐ถ to the(extension of the) ๐ต๐ถ1 side has the length ๐‘ฅ shown in Fig. 6.27(b). So the areaof triangle ๐ถ๐ต๐ถ1 is (1/2)๐‘โ„Ž = (1/2)๐‘ ยท ๐‘ฅ = ๐‘๐‘ฅ/2, which is indeed half the areaof the ๐‘…1 rectangle with sides ๐‘ and ๐‘ฅ.

The preceding two paragraphs, combined with the fact that triangles ๐ด1๐ต๐ด and๐ถ๐ต๐ถ1 are congruent, tell us that ๐‘Ž2/2 = ๐‘๐‘ฅ/2, which (after multiplying bothsides by 2) means that ๐‘Ž2 = ๐‘๐‘ฅ. So the ๐‘๐‘ฅ area of the ๐‘…1 rectangle is simply ๐‘Ž2.

We can now repeat (or rather, just imagine repeating) the entire process above,but with the light-shaded congruent triangles. The result will be that the area ofthe ๐‘…2 rectangular part of the square with side ๐‘ that is below/right of the dashedline equals ๐‘2.

Finally, since the ๐‘Ž2 and ๐‘2 areas of the ๐‘…1 and ๐‘…2 rectangles add up to the ๐‘2

area of the square with side ๐‘, we conclude that ๐‘Ž2 + ๐‘2 = ๐‘2, as desired.

The above proof might seem long, but hereโ€™s the quick summary: Triangles๐ด1๐ต๐ด and ๐ถ๐ต๐ถ1 are congruent because they can be rotated into each other.So they have the same area. Fig. 6.27 then shows that these triangles have(with appropriately chosen bases) the same heights as the ๐‘Ž2 square and the ๐‘…1rectangle. This square and rectangle therefore also have the same area (bothtwice the common triangle area). So the ๐‘…1 rectangle has area ๐‘Ž2. Likewise, the๐‘…2 rectangle has area ๐‘2. Finally, the ๐‘…1 and ๐‘…2 areas add up to ๐‘2.

15. In terms of ๐‘…, plugging โ„Ž = ๐‘…/16 into Eqs. (6.21) and (6.23) gives

๐‘‘exact =โˆš

2๐‘…โ„Ž + โ„Ž2 =โˆš

2๐‘…(๐‘…/16) + (๐‘…/16)2

=โˆš๐‘…2(1/8 + 1/256) = ๐‘…

โˆš0.1289 = (0.3590)๐‘…,

๐‘‘approx =โˆš

2๐‘…โ„Ž =โˆš

2๐‘…(๐‘…/16)=

โˆš๐‘…2(1/8) = ๐‘…

โˆš0.125 = (0.3536)๐‘…. (6.67)

With ๐‘… = 6,400 km, these results become

๐‘‘exact = (0.3590)(6,400 km) = 2298 km,

๐‘‘approx = (0.3536)(6,400 km) = 2263 km. (6.68)

The difference in these answers is only 35 km, which is about 0.015 (equivalently,1.5%) of the exact 2298 km distance. So the approximation in Eq. (6.23) is stillvery good, even for the large (but still small compared with ๐‘…) โ„Ž value of theSpace Station.

6.7. Exercise solutions 299

R โˆ’ h

hBostonNYC

R

d/2d/2

Figure 6.28

16. The setup is shown in Fig. 6.28. The desired maximum depth is โ„Ž, and the totaldistance from Boston to NYC is ๐‘‘ = 300 km.

There is technically an ambiguity about whether the 300 km is the straight-linedistance or the curved distance along the surface of the earth. But it doesnโ€™tmatter since these two distances are essentially the same. If the two cities weresignificantly farther apart, then we would have to worry about this issue. Thestraight-line distance is the useful one in Fig. 6.28, so if (as would be the casein real life) weโ€™re given the curved distance, the first thing weโ€™d have to dois somehow calculate the straight-line distance. But thereโ€™s no need to do thathere, since the two distances are indistinguishable (because 300 km is sufficientlysmall compared with the 6,400 km radius of the earth). Itโ€™s possible to show (withtrigonometry; see Chapter 16) that the two distances differ by only about 0.01%.

The right triangle in Fig. 6.28 has legs ๐‘‘/2 and ๐‘… โˆ’ โ„Ž, and hypotenuse ๐‘…. So thePythagorean theorem gives

(๐‘‘/2)2 + (๐‘… โˆ’ โ„Ž)2 = ๐‘…2 =โ‡’ (๐‘‘/2)2 = ๐‘…2 โˆ’ (๐‘… โˆ’ โ„Ž)2

=โ‡’ ๐‘‘2/4 =๏ฟฝ๏ฟฝ๐‘…2 โˆ’ (๏ฟฝ๏ฟฝ๐‘…2 โˆ’ 2๐‘…โ„Ž + โ„Ž2). (6.69)

As we did in the distance-to-horizon problem in the text, we can ignore the โ„Ž2

term, because it is negligible compared with the 2๐‘…โ„Ž term. Weโ€™re then left with

๐‘‘2

4= 2๐‘…โ„Ž =โ‡’ โ„Ž =

๐‘‘2

8๐‘…, (6.70)

where we have divided both sides by 2๐‘… (and then switched sides). Plugging inthe Boston-NYC distance of 300 km gives

โ„Ž =(300 km)2

8(6,400 km) โ‰ˆ 1.75 km โ‰ˆ 1.1 miles. (6.71)

Is this answer larger or smaller than what you expected? Personally, my firstguess was that the tunnel would be deeper than this. But in retrospect, the earthis nearly flat on the scale of 300 km, so this small value of โ„Ž is quite believable.

300 Chapter 6. Pythagorean theorem

Note that our earlier result for the tower height โ„Ž in Eq. (6.28) is 4 times theresult for the depth โ„Ž in Eq. (6.70). So if you build a tower in Boston tall enoughto see NYC (the ground there, not just the skyline), and if you also dig a tunnelbetween the two cities, then the tower will be 4 times as tall as the tunnel isdeep. Since we just found that the tunnel will be 1.75 km deep, the tower will be4(1.75 km) = 7 km (or 4.3 miles) tall.

17. The setup is shown in Fig. 6.29. In the bottom face of the box, we have a righttriangle with legs ๐‘Ž and ๐‘, so our first application of the Pythagorean theoremgives the hypotenuse ๐‘‘1 as

๐‘Ž2 + ๐‘2 = ๐‘‘21 =โ‡’ ๐‘‘1 =

โˆš๐‘Ž2 + ๐‘2. (6.72)

This is the length of a diagonal of the bottom (or top) face of the box, not thewhole box itself.

a

d1

d2

b

c

Figure 6.29

We now note that we have another right triangle โ€“ the vertical one with legs ๐‘‘1and ๐‘, and hypotenuse ๐‘‘2. So our second application of the Pythagorean theoremgives

๐‘‘21 + ๐‘2 = ๐‘‘2

2 . (6.73)

Substituting the value of ๐‘‘21 from Eq. (6.72) into this equation gives

(๐‘Ž2 + ๐‘2) + ๐‘2 = ๐‘‘22 =โ‡’ ๐‘‘2 =

โˆš๐‘Ž2 + ๐‘2 + ๐‘2. (6.74)

This is the desired length of the diagonal of the box. This expression is symmetricin ๐‘Ž, ๐‘, and ๐‘, because (just as with the Pythagorean theorem) it canโ€™t matterwhich of the three dimensions you arbitrarily choose to label as ๐‘Ž, or ๐‘, or๐‘. Said in another way, if your first application of the Pythagorean theoreminstead involved one of the side faces, you would still end up with the same final๐‘‘2 =

โˆš๐‘Ž2 + ๐‘2 + ๐‘2 result.

6.7. Exercise solutions 301

18. The key is to unroll the cylinder into a flat rectangle, as shown in Fig. 6.30. Thewidth of the rectangle is the ๐œ‹๐‘‘ circumference of the cylinder, and the diagonal๐ฟ is the desired length of the staircase. (Yes, the sides in Fig. 6.30 are in factin the correct proportion, for the given cylinder shown in Fig. 6.15. The ๐œ‹๐‘‘

circumference is longer than you might think.)

h

L

ฯ€d

Figure 6.30

Applying the Pythagorean theorem to the right triangle in the figure gives thelength ๐ฟ of the staircase as

(๐œ‹๐‘‘)2 + โ„Ž2 = ๐ฟ2 =โ‡’ ๐ฟ =โˆš๐œ‹2๐‘‘2 + โ„Ž2. (6.75)

This is the answer in terms of the general lengths ๐‘‘ and โ„Ž. But in the special casewhere ๐‘‘ = โ„Ž, we have

๐ฟ =โˆš๐œ‹2โ„Ž2 + โ„Ž2 =

โˆš(๐œ‹2 + 1)โ„Ž2 =

โˆš๐œ‹2 + 1 ยท โ„Ž โ‰ˆ (3.3)โ„Ž. (6.76)

We see that the length ๐ฟ of the staircase is only slightly longer than the circum-ference ๐œ‹๐‘‘. This is because the long leg of the right triangle, ๐œ‹โ„Ž, is significantlylonger than the short leg, โ„Ž.We can check our general result in Eq. (6.75) for some special cases. If โ„Ž = 0,then the cylinder has zero height, so the โ€œstaircaseโ€ is just a horizontal circle. AndEq. (6.75) correctly gives the

โˆš๐œ‹2๐‘‘2 + 02 = ๐œ‹๐‘‘ circumference of the circle. In the

other extreme, if ๐‘‘ = 0, the cylinder is just a vertical segment, so the โ€œstaicraseโ€is a vertical ladder. And Eq. (6.75) correctly gives the

โˆš๐œ‹2 ยท 02 + โ„Ž2 = โ„Ž height

of the segment.

19. If the pendulum is presently ๐‘ฆ above the lowest point, then it is ๐‘… โˆ’ ๐‘ฆ below thepivot (the center of the circular arc), as shown in Fig. 6.31. So the sides of theright triangle in the figure are ๐‘ฅ, ๐‘… โˆ’ ๐‘ฆ, and ๐‘…. The Pythagorean theorem thengives

๐‘ฅ2 + (๐‘… โˆ’ ๐‘ฆ)2 = ๐‘…2 =โ‡’ ๐‘ฅ2 = ๐‘…2 โˆ’ (๐‘… โˆ’ ๐‘ฆ)2

=โ‡’ ๐‘ฅ2 =๏ฟฝ๏ฟฝ๐‘…2 โˆ’ (๏ฟฝ๏ฟฝ๐‘…2 โˆ’ 2๐‘…๐‘ฆ + ๐‘ฆ2)

=โ‡’ ๐‘ฅ2 = 2๐‘…๐‘ฆ โˆ’ ๐‘ฆ2. (6.77)

302 Chapter 6. Pythagorean theorem

R โˆ’ y

y

x

R

Figure 6.31

Weโ€™ll now make the approximation where we ignore the ๐‘ฆ2 term since it is muchsmaller than the 2๐‘…๐‘ฆ term. (๐‘ฆ2 is ๐‘ฆ/2๐‘… times 2๐‘…๐‘ฆ, and this factor of ๐‘ฆ/2๐‘… issmall, since weโ€™re assuming that ๐‘ฆ is much smaller than ๐‘….) Weโ€™re therefore leftwith

๐‘ฅ2 โ‰ˆ 2๐‘…๐‘ฆ =โ‡’ ๐‘ฆ โ‰ˆ ๐‘ฅ2

2๐‘…. (6.78)

This problem is just an upside-down version of the tunnel setup in Exercise 6.16,with ๐‘‘/2 replaced with ๐‘ฅ, and โ„Ž replaced with ๐‘ฆ. The pendulum moves in thearc of a circle with radius ๐‘…, just like the surface of the earth took the shape of acircle with radius ๐‘… in Exercise 6.16.

The ๐‘ฅ2 in Eq. (6.78) means that the pendulumโ€™s motion takes (approximately,assuming ๐‘ฆ is much smaller than ๐‘…) the form of a parabola. Parabolas arefunctions of the form ๐‘ฆ = ๐ด๐‘ฅ2. Weโ€™ll talk about these, along with other types offunctions, in Chapter 8. What weโ€™ve shown in this exercise is that a circle lookslike a parabola, at least near the bottom point. The circle/parabola starts out flatand then gradually gets steeper. If you double the ๐‘ฅ value from, say, ๐‘Ž to 2๐‘Ž,then the ๐‘ฆ value quadruples from ๐‘Ž2/2๐‘… to (2๐‘Ž)2/2๐‘… = 4 ยท ๐‘Ž2/2๐‘…. Similarly,increasing ๐‘ฅ by a factor of 10 increases ๐‘ฆ by a factor of 102 = 100 (assuming ๐‘ฆ

is still much smaller than ๐‘…).

20. Since weโ€™ve chosen the radii of the large circles be 1, the hypotenuse of the righttriangle has length 1 + ๐‘Ÿ. And the vertical leg has length 1 โˆ’ ๐‘Ÿ, because its topand bottom ends are at heights of, respectively, 1 and ๐‘Ÿ above the bottom line.The right triangle therefore has legs 1 and 1 โˆ’ ๐‘Ÿ, and hypotenuse 1 + ๐‘Ÿ.

Since the side lengths of 1โˆ’ ๐‘Ÿ, 1, and 1 + ๐‘Ÿ are equally spaced (with the commonspacing being ๐‘Ÿ), we can simply invoke the result from Example 6.3, which tellsus that the sides are in the ratio of 3 : 4 : 5, as desired. So weโ€™re done. But letโ€™swork it out again anyway. The Pythagorean theorem applied to the right triangle

6.7. Exercise solutions 303

in Fig. 6.17 gives

1 + (1 โˆ’ ๐‘Ÿ)2 = (1 + ๐‘Ÿ)2 =โ‡’ 1 = (1 + ๐‘Ÿ)2 โˆ’ (1 โˆ’ ๐‘Ÿ)2

=โ‡’ 1 = (AA1 + 2๐‘Ÿ + ๏ฟฝ๏ฟฝ๐‘Ÿ2) โˆ’ (AA1 โˆ’ 2๐‘Ÿ + ๏ฟฝ๏ฟฝ๐‘Ÿ2)=โ‡’ 1 = 4๐‘Ÿ =โ‡’ ๐‘Ÿ = 1/4. (6.79)

The legs of the right triangle are then 1 โˆ’ ๐‘Ÿ = 3/4, and 1. And the hypotenuse is1 + ๐‘Ÿ = 5/4. Scaling all of these up by a factor of 4 gives sides of 3, 4, and 5, asdesired.

If you instead start with a general radius ๐‘… for the large circles, instead of 1, youwill (as you can verify) end up with sides of 3๐‘…/4, ๐‘…, and 5๐‘…/4, which are againin the ratio of 3 : 4 : 5. The value of ๐‘Ÿ is different (๐‘…/4 instead of 1/4), but the3 : 4 : 5 ratio isnโ€™t affected.

21. This exercise is very similar to Example 6.2. Fig. 6.32 shows a 30โ—ฆ pie piece(just the triangle, without the rounded end). Letting the radius be 1 as usual, the30-60-90 triangle in the left part of the pie piece has legs with lengths 1/2 andโˆš

3/2 (from Section 5.4), as shown.

2

s

1

30

60

3/23/

2/

1 โˆ’

1

Figure 6.32

Since the bottom side of the pie piece has length 1 (because itโ€™s also a radius),a length 1 โˆ’

โˆš3/2 is left for the short segment on the right side, as shown. The

Pythagorean theorem applied to the right triangle in the right part of the pie piecethen gives the dodecagonโ€™s side length ๐‘  as

๐‘ 2 = (1/2)2 +(1 โˆ’

โˆš3/2

)2 = 1/4 +(1 โˆ’

โˆš3 + 3/4

)= 2 โˆ’

โˆš3. (6.80)

So ๐‘  =โˆš

2 โˆ’โˆš

3 โ‰ˆ 0.518. Multiplying this by 12 to find the perimeter ofthe dodecagon gives ๐‘ƒdodec โ‰ˆ 6.21. The ๐ถcirc > ๐‘ƒdodec statement that thecircumference of the circle is greater than the perimeter of the dodecagon is then2๐œ‹ > 6.21, or equivalently ๐œ‹ > 3.1, after dividing by 2. This value is about 99%of the true ๐œ‹ โ‰ˆ 3.14 value, so the approximation is a very good one.

304 Chapter 6. Pythagorean theorem

22. (a) The upper-left right triangle in Fig. 6.18 has hypotenuse 1 and vertical leg๐‘Ž๐‘›/2. So the Pythagorean theorem gives the horizontal leg ๐‘ฅ as

๐‘ฅ2 + (๐‘Ž๐‘›/2)2 = 12 =โ‡’ ๐‘ฅ2 = 1 โˆ’ ๐‘Ž2๐‘›/4 =โ‡’ ๐‘ฅ =

โˆš1 โˆ’ ๐‘Ž2

๐‘›/4. (6.81)

Since ๐‘ฅ + ๐‘ฆ equals the radius 1, we have ๐‘ฆ = 1 โˆ’ ๐‘ฅ = 1 โˆ’โˆš

1 โˆ’ ๐‘Ž2๐‘›/4.

We can now use the upper-right right triangle in Fig. 6.18 to solve for thehypotenuse ๐‘Ž2๐‘›. The legs are ๐‘Ž๐‘›/2 and ๐‘ฆ (which we just found), so thePythagorean theorem gives

๐‘Ž22๐‘› = (๐‘Ž๐‘›/2)2 + ๐‘ฆ2

= (๐‘Ž๐‘›/2)2 +(1 โˆ’

โˆš1 โˆ’ ๐‘Ž2

๐‘›/4)2

=๏ฟฝ๏ฟฝ๏ฟฝ๐‘Ž2๐‘›/4 +

(1 โˆ’ 2

โˆš1 โˆ’ ๐‘Ž2

๐‘›/4 + (1 โˆ’๏ฟฝ๏ฟฝ๏ฟฝ๐‘Ž2๐‘›/4)

)= 2 โˆ’ 2

โˆš1 โˆ’ ๐‘Ž2

๐‘›/4

=โ‡’ ๐‘Ž2๐‘› =

โˆš2 โˆ’ 2

โˆš1 โˆ’ ๐‘Ž2

๐‘›/4 , (6.82)

as desired. This isnโ€™t the cleanest answer, but as least itโ€™s an answer. If weknow ๐‘Ž๐‘›, we just need to plug it into this expression, and ๐‘Ž2๐‘› pops out.

(b) Table 6.3 shows the ๐‘Ž๐‘› values for various ๐‘›โ€™s (powers of 2), along with theresulting estimate of ๐œ‹ (a lower bound), and also the ratio of this estimate tothe true value of ๐œ‹. The perimeter of the ๐‘›-gon is ๐‘› ยท๐‘Ž๐‘›, so the๐ถcirc > ๐‘ƒ๐‘›-gonstatement is 2๐œ‹ ยท 1 > ๐‘›๐‘Ž๐‘› =โ‡’ ๐œ‹ > ๐‘›๐‘Ž๐‘›/2. This is the estimate of ๐œ‹ in thethird column. Each ๐‘Ž๐‘› in the table is obtained from the preceding oneby plugging that one into Eq. (6.82). Note that the ๐‘Ž8 =

โˆš2 โˆ’

โˆš2 value

correctly agrees with the result in Example 6.2.

๐‘› ๐‘Ž๐‘› ๐‘›๐‘Ž๐‘›/2 (๐‘›๐‘Ž๐‘›/2)/๐œ‹2 2 2 0.644

โˆš2 2

โˆš2 = 2.83 0.90

8โˆš

2 โˆ’โˆš

2 4โˆš

2 โˆ’โˆš

2 = 3.06 0.97416 0.39018064 3.1214 0.993632 0.19603428 3.13655 0.998464 0.09813535 3.14033 0.99960128 0.04908246 3.141277 0.99990256 0.02454308 3.141514 0.999975

Table 6.3: Estimates of ๐œ‹ using ๐‘›-gons, where ๐‘› is a power of 2

6.7. Exercise solutions 305

It is in fact legal to start the table with the ๐‘› = 2 case, as we have done. Ofcourse, a 2-sided polygon isnโ€™t much of a polygon. Itโ€™s the back-and-forthdiameter we encountered in Exercise 5.23, so it has โ€œsidesโ€ of ๐‘Ž2 = 2.Plugging this into Eq. (6.82) correctly gives the ๐‘Ž4 =

โˆš2 side of a square.

However, if youโ€™re uncomfortable using the ๐‘› = 2 case, you can simply startthe table with the ๐‘› = 4 case.

Remarks: Itโ€™s possible to write the ๐‘Ž๐‘› lengths in terms of square roots for๐‘› = 16 and higher, but the expressions become long and tedious. So weopted to use the (approximate) decimal forms in the table. In most cases, wedidnโ€™t actually need to keep as many digits as we did, as far as a specific ๐‘Ž๐‘›is concerned. However, the accuracy of any given ๐‘Ž๐‘› affects the accuracyof higher ๐‘Ž๐‘›โ€™s. And the higher the ๐‘›, the more digits we need to know. Forexample, for ๐‘› = 16 our ๐‘›๐‘Ž๐‘›/2 estimate for ๐œ‹ differs from the actual valueof ๐œ‹ (โ‰ˆ 3.14159) in the second digit after the decimal point, whereas for๐‘› = 256 it differs in the fifth digit.Fig. 6.33 shows a 32-gon, which is very close to a smooth circle. The0.9984 (equivalently, 99.84%) ratio in Table 6.3 for ๐‘› = 32 seems quitereasonable, since the perimeter of the 32-gon is essentially equal to thecircumference of the circle in which it is inscribed. We havenโ€™t drawn thecircle, because it would be nearly indistinguishable from the 32-gon.

(n = 32)

Figure 6.33

An inspection of Table 6.3 shows that the error (the difference from 1) inthe (๐‘›๐‘Ž๐‘›/2)/๐œ‹ value in the last column decreases by a factor of 4 from one ๐‘›to the next (except for the first few small values of ๐‘›). For example, 0.9984differs from 1 by 0.0016, and then 0.99960 differs from 1 by 0.0004, andthen 0.99990 differs from 1 by 0.0001. Each of these differences is 1/4of the previous one. To prove that this pattern holds in general, we wouldneed more machinery than we have at our disposal, so weโ€™ll just accept itas an interesting fact here.

306 Chapter 6. Pythagorean theorem

When producing a ๐œ‹ estimation,Use ๐‘›-gons without hesitation,Because doubling your ๐‘›,Yet again, and again,Yields improvement with each iteration!

You can also make another table of doubled ๐‘› values, but now starting with๐‘› = 3. The ๐‘› values will be 3, 6, 12, 24, 48, and so on. From Exercise 5.23,the ๐‘Ž3 side of an equilateral triangle (assuming a circle radius of 1, as usual)is ๐‘Ž3 =

โˆš3. You can quickly show that plugging this into Eq. (6.82) correctly

gives the ๐‘Ž6 = 1 side of a hexagon. And then plugging this into Eq. (6.82)reproduces the ๐‘Ž12 =

โˆš2 โˆ’

โˆš3 dodecagon result from Exercise 6.21. โ™ฃ

23. (a) The two sub-triangles in Fig. 6.34 are indeed isosceles, because they eachhave two sides equal to the radius ๐‘Ÿ . Let the angle at ๐ด be ๐›ผ. Then becausetriangle ๐ด๐ท๐ถ is isosceles, we have the other angle ๐›ผ shown.

C

Ar

ฮฑ ฮฒ

ฮฒฮฑ

r

r

BD

Figure 6.34

Likewise, let the angle at ๐ต be ๐›ฝ. Then because triangle ๐ต๐ท๐ถ is isosceles,we have the other angle ๐›ฝ shown. The three angles in the overall triangleare then โˆ ๐ด = ๐›ผ, โˆ ๐ต = ๐›ฝ, and โˆ ๐ถ = ๐›ผ + ๐›ฝ. The sum of these angles mustbe 180โ—ฆ, so

๐›ผ + ๐›ฝ + (๐›ผ + ๐›ฝ) = 180โ—ฆ =โ‡’ 2๐›ผ + 2๐›ฝ = 180โ—ฆ

=โ‡’ ๐›ผ + ๐›ฝ = 90โ—ฆ. (6.83)

The lefthand side of this last relation is simply โˆ ๐ถ, so we have shown thatโˆ ๐ถ = 90โ—ฆ, as desired. This 90โ—ฆ result for โˆ ๐ถ is a special case of a moregeneral theorem weโ€™ll prove in Chapter 11.

Remark: The above โˆ ๐ถ = 90โ—ฆ result provides a quick answer to thequestion of how to classify all the different possible shapes of rectanglesthat have a specified length ๐ฟ for their diagonal. We can do this by drawing

6.7. Exercise solutions 307

a circle with diameter ๐ฟ. If we draw the horizontal diameter, along withany other arbitrary diameter (the dashed lines in Fig. 6.35), weโ€™ll endup with a rectangle, because the result of this exercise tells us that thehorizontal diameter leads to two right angles (the solid little boxes shown),and the titled dashed diameter leads to two others (the dashed little boxes).Depending on how tilted the dashed diameter is, we can produce a thinrectangle like the one on the left, or the โ€œfatโ€ square on the right.

L L

Figure 6.35

Exercise 6.4 dealt with a computer screen with a 13.3 inch (or 17 inch)diagonal. We now see how to generate all possible rectangular screens witha 13.3 inch diagonal. However, probably no one is going to want to use ascreen as squat as the left rectangle in Fig. 6.35. In the old days, screenswere more square-ish, but theyโ€™ve gotten wider (although not as wide asthe left one in Fig. 6.35) as the years have gone by. โ™ฃ

(b) In Fig. 6.36, let the sides of right triangle ๐ด๐ต๐ถ be 2๐‘Ž, 2๐‘, and 2๐‘, forconvenience (so that we wonโ€™t have a bunch a 1/2 factors floating around).Let ๐ท be the midpoint of the hypotenuse, so that ๐ต๐ท = ๐ท๐ด = ๐‘. Draw thevertical segment ๐ท๐ธ . Then triangle ๐ด๐ท๐ธ is similar to triangle ๐ด๐ต๐ถ (theyboth have a right angle along with the common angle at ๐ด, which meansthat their third angles are also the same), and it is half a large (since its๐ด๐ท = ๐‘ hypotenuse is half the ๐ด๐ต = 2๐‘ hypotenuse). So ๐ท๐ธ = ๐‘Ž (half of๐ต๐ถ), and ๐ด๐ธ = ๐‘ (half of ๐ด๐ถ). This leaves ๐‘ for ๐ถ๐ธ , as shown. From thePythagorean theorem applied to triangles ๐ด๐ท๐ธ and ๐ถ๐ท๐ธ , the lengths of๐ด๐ท and ๐ถ๐ท are both

โˆš๐‘Ž2 + ๐‘2 (which equals ๐‘). So they are equal (and

hence also equal to ๐ท๐ต), as desired. ๐ท is therefore the center of the circlepassing through ๐ด, ๐ต, and ๐ถ. And since any chord of a circle containingthe center is a diameter, we see that ๐ด๐ต is a diameter, as we wanted to show.To succinctly summarize the results in parts (a) and (b) of this exercise:If a triangle is inscribed in a circle, then (a) If one side is a diameter,then the triangle is right, and (b) If the triangle is right, then one side

308 Chapter 6. Pythagorean theorem

B

C

2a

A

D

Eb b

c

ca

Figure 6.36

(the hypotenuse) is a diameter. These two statements are the converses(reverses) of each other.

Remark: We just showed in part (b) that for a given hypotenuse length,the midpoint of the hypotenuse is always the same distance (half the lengthof the hypotenuse) from the right angle, no matter what the lengths of thelegs are. It doesnโ€™t matter if the right triangle is a โ€œfatโ€ 45-45-90 one,or a thin 1-89-90 one. As long as the hypotenuse has a fixed length, thedistance from the midpoint to the right angle is always the same (half thehypotenuse).This fact is relevant to the famous โ€œsliding ladderโ€ problem. A ladder slidesdown a wall, as shown in Fig. 6.37, always maintaining contact with thewall and the floor. What is the path taken by the midpoint? Since themidpoint of the ladder (the midpoint of the hypotenuse; the dots shown) isalways the same distance from the corner, we see that the midpoint tracesout a quarter circle, as shown. โ™ฃ

24. (a) First proof: Weโ€™ll present two proofs. The first uses a symmetry argument,which is a standard (and slick) method of proof. Fig. 6.38(a) shows a circlesitting on top of a line; the line is tangent to the circle. This setup hasleft/right symmetry, meaning that if we flip it over (so that left and rightare reversed), it looks the same. Equivalently, the mirror image looks thesame. Said in yet another way, it looks the same if we view it through theback of the paper.If we flip over the setup in Fig. 6.38(a), it turns into Fig. 6.38(b), whichmean that the ๐›ผ and ๐›ฝ angles have switched; ๐›ผ is now on the right, and ๐›ฝ

is on the left. However, the above left/right symmetry property tells us thatFig. 6.38(b) must be exactly the same setup as Fig. 6.38(a), which meansthat the ๐›ผ angle must still be on the left, and likewise the ๐›ฝ angle must stillbe on the right. Putting the preceding two sentences together, we see that

6.7. Exercise solutions 309

wall

floor

Figure 6.37

A A

ฮฑ ฮฑฮฒ ฮฒ

B B

flip over

(a) (b)

Figure 6.38

the left angle in Fig. 6.38(b) must be equal to both ๐›ผ (from the precedingsentence) and also ๐›ฝ (from two sentences ago). We therefore conclude that๐›ผ = ๐›ฝ. And since these two equal angles add up to 180โ—ฆ (since togetherthey form a straight line), they must each be 90โ—ฆ, as we wanted to show.

Second proof: For this second proof, weโ€™ll use the Pythagorean theoremin what is called a proof by contradiction. In a proof by contradiction,if weโ€™re trying to prove a particular statement (letโ€™s label the statement asโ€œ๐‘ƒโ€), the strategy is to make the assumption that ๐‘ƒ is not true (which is theopposite of what weโ€™re actually trying to show). The goal is to then showthat this assumption leads to a false statement. It then logically follows thatour not-๐‘ƒ assumption must have been incorrect, because true things canโ€™tlogically lead to false things. So our original ๐‘ƒ statement must be correct(because if not-๐‘ƒ is false, then ๐‘ƒ must be true), as we wanted to show.For example, consider the statement, โ€œThere are no integers (positive ornegative) ๐‘Ž and ๐‘ for which 2๐‘Ž + 6๐‘ = 5.โ€ Now, thereโ€™s no chance oftesting all of the infinite number of possible ๐‘Ž and ๐‘ values and showing

310 Chapter 6. Pythagorean theorem

that none of them work. So we need a different method of proof, and aproof by contradiction works well here. In search of a contradiction, letโ€™sassume that there do exist integers ๐‘Ž and ๐‘ for which 2๐‘Ž + 6๐‘ = 5. It thenfollows (by dividing both sides by 2) that there exist integers ๐‘Ž and ๐‘ forwhich ๐‘Ž + 3๐‘ = 5/2. But this is a false statement, because the lefthand sideis an integer (if ๐‘Ž and ๐‘ are integers), whereas the righthand side is not.Our initial assumption (that there do exist integers. . . ) must therefore havebeen incorrect, which means that we have successfully proved that thereare no integers ๐‘Ž and ๐‘ for which 2๐‘Ž + 6๐‘ = 5. Weโ€™ll discuss proofs bycontradiction in more detail in Chapter 12.

In our present tangent-line problem, our assumption (which weโ€™ll end upshowing is incorrect) is that the tangent line is not perpendicular to theradius at the point of contact. If this assumption is true, then of the twoangles the radius makes with the tangent, one must be larger than 90โ—ฆ, andone must be smaller than 90โ—ฆ, as shown in Fig. 6.39. (Donโ€™t try to make toomuch sense of this figure, since weโ€™re going to show that it canโ€™t actuallylook this way.)

radius (R)

circle

tangent

smaller than 90A

center

Figure 6.39

Consider the angle that is smaller than 90โ—ฆ, and draw a right trianglecontaining that angle, as shown. Since the leg of a right triangle is (due tothe Pythagorean theorem) always shorter than the hypotenuse, which is ๐‘…

here, the vertical leg of our right triangle is smaller than ๐‘…. This impliesthat point ๐ด must be inside the circle (because its distance from the centeris less than the radius ๐‘…). This contradicts the fact that every point on atangent line lies outside the circle (except for the single point that lies on thecircle). Therefore, since our assumption of non-perpendicularity leads to afalse statement (that ๐ด is inside the circle), we conclude that the assumptionmust have been incorrect. The radius and tangent line must therefore in factbe perpendicular.

6.7. Exercise solutions 311

(b) First proof: Weโ€™ll again present two proofs. As in part (a), the firstone uses a symmetry argument, and the second one uses the Pythagoreantheorem (but isnโ€™t a proof by contradiction). In Fig. 6.40(a), the two tangentsare drawn from a point directly above the center of the circle. As with thesetup in part (a), this system has left/right symmetry. So if we flip it over(or look at it in a mirror, or through the back of the paper), we obtain theidentical system in Fig. 6.40(b). The length ๐‘‘1 is now on the right. Butsince itโ€™s the same setup, the length ๐‘‘2 must also be on the right. Hence๐‘‘1 = ๐‘‘2, as desired. This is exactly the same reasoning that led to the ๐›ผ = ๐›ฝ

conclusion in part (a).

(a)

d2 d2d1 d1

(b)

Figure 6.40

Second proof: From part (a), we know that the tangents are perpendicularto the radii where they touch, as shown in Fig. 6.41. The two right trianglesshown have a common hypotenuse ๐ฟ, along with a common leg (the radius๐‘…). So the Pythagorean theorem tells us that the other legs have the samelength (both equal to

โˆš๐ฟ2 โˆ’ ๐‘…2), as desired.

R R

P

L

Figure 6.41

25. Eq. (6.35) in Example 6.4 tells us that ๐ต๐ด = 2โˆš๐‘Ž๐‘. And since Eq. (6.35) is

valid for any two circles that touch tangentially, we can also apply it to the lefttwo circles in Fig. 6.21, with radii ๐‘ and ๐‘Ÿ. ๐ต and ๐ถ are now the relevant pointsof contact on the tangent line, so Eq. (6.35) tells us that ๐ต๐ถ = 2

โˆš๐‘Ÿ๐‘. Likewise,

applying Eq. (6.35) to the right two circles in Fig. 6.21, with radii ๐‘Ÿ and ๐‘Ž, gives๐ถ๐ด = 2

โˆš๐‘Ÿ๐‘Ž.

312 Chapter 6. Pythagorean theorem

We can now use the fact that ๐ต๐ถ + ๐ถ๐ด = ๐ต๐ด, which gives

2โˆš๐‘Ÿ๐‘ + 2

โˆš๐‘Ÿ๐‘Ž = 2

โˆš๐‘Ž๐‘ =โ‡’

โˆš๐‘Ÿ(โˆš

๐‘ +โˆš๐‘Ž)=โˆš๐‘Ž๐‘

=โ‡’โˆš๐‘Ÿ =

โˆš๐‘Ž๐‘

โˆš๐‘Ž +

โˆš๐‘, (6.84)

where we have divided both sides by 2, factored out theโˆš๐‘Ÿ on the lefthand side,

and then divided both sides byโˆš๐‘Ž +

โˆš๐‘. Squaring both sides then gives

๐‘Ÿ =

( โˆš๐‘Ž๐‘

โˆš๐‘Ž +

โˆš๐‘

)2

=๐‘Ž๐‘

(โˆš๐‘Ž +โˆš๐‘)2

. (6.85)

Note that in the special case where ๐‘ = ๐‘Ž, we have (replacing every ๐‘ in Eq. (6.85)with ๐‘Ž)

๐‘Ÿ =๐‘Ž ยท ๐‘Ž

(2โˆš๐‘Ž)2 =๐‘Ž2

4๐‘Ž=๐‘Ž

4. (6.86)

So the small circle has 1/4 the radius of the large circles. This agrees with theresult in Exercise 6.20, where we solved the ๐‘Ž = ๐‘ case directly (with ๐‘Ž = ๐‘ = 1).

26. As in Example 6.4, the short leg of the shaded triangle is the ๐‘Ž โˆ’ ๐‘ difference ofthe radii. But the hypotenuse here isnโ€™t ๐‘Ž + ๐‘, as it was in Example 6.4. Instead,it equals

โˆš๐‘Ž2 + ๐‘2 because it is the hypotenuse of right triangle ๐ด๐ต๐ถ. This is

indeed a right triangle, because the dashed tangent is perpendicular to the radiusof the left circle at ๐ถ, from Exercise 6.24(a).

The long leg of the shaded triangle is the desired distance ๐‘‘, so applying thePythagorean theorem to the shaded triangle gives

๐‘‘2 + (๐‘Ž โˆ’ ๐‘)2 =(โˆš

๐‘Ž2 + ๐‘2)2

=โ‡’ ๐‘‘2 =(โˆš

๐‘Ž2 + ๐‘2)2 โˆ’ (๐‘Ž โˆ’ ๐‘)2

= (๏ฟฝ๏ฟฝ๐‘Ž2 +@@๐‘2) โˆ’ (๏ฟฝ๏ฟฝ๐‘Ž2 โˆ’ 2๐‘Ž๐‘ +@@๐‘

2)= 2๐‘Ž๐‘

=โ‡’ ๐‘‘ =โˆš

2๐‘Ž๐‘ =โˆš

2โˆš๐‘Ž๐‘ = (1.41)

โˆš๐‘Ž๐‘. (6.87)

Thisโˆš

2โˆš๐‘Ž๐‘ = (1.41)

โˆš๐‘Ž๐‘ answer is smaller than the 2

โˆš๐‘Ž๐‘ answer in Exam-

ple 6.4. This makes sense, because the centers of the circles are closer togetherin this exercise (the circles partially overlap here).

In the special case where ๐‘Ž = ๐‘, we obtain ๐‘‘ =โˆš

2โˆš๐‘Ž2 =

โˆš2๐‘Ž. In this case, the

๐ต๐ถ and ๐ด๐ถ segments are two sides of a square, with side length ๐‘Ž. The distance๐‘‘ is the same as the diagonal ๐ต๐ด of this square (the distance between the centersof the circles), as you can verify by drawing a picture. And the diagonal of asquare with side ๐‘Ž correctly has length

โˆš2๐‘Ž.